Download as pdf or txt
Download as pdf or txt
You are on page 1of 104

1.

A 23 year old woman with complex partial seizures was Initially treated with phenytoin, then
switched to carbamazepine, and is currently on levetiracetam after failing the first two therapies. After
a seizure-free period of six months upon starting levetiracetam, she now has recurrence of 2-3
seizures/month. What is the most appropriate next step in her management?
A. Evaluation for vagus nerve stimulation.
B. Add-on therapy with oxcarbazepine.
C. Switch to therapy with oxcarbazepine.
D Evaluation for resective epilepsy surgery
The patient meets the criteria for drug resistant epilepsy (Kwan et al., 201 0), and is unlikely to
become seizurefree with further medication trials. The most appropriate next step In her management
is evaluation for resective epilepsy surgery. In a single-institution, prospective, cohort study of 780
adolescents and adults prescribed their first AED from 1982-2001, 50.4%, 10.7%, and 2.7% percent
of patients became seizure free for at least one year with the first, second and third treatment
schedules, respectively (Mohanraj and Brodie 2006). Some studies show rates of seizure freedom in
chronic epilepsy as high as approximately 15%, depending on trial design, how an adequate trial of
AED is defined, or use of newer AEDs. However, this is still significantly lower than chances of
seizure freedom if she is a candidate for resective epilepsy surgery. Therefore, mono therapy or add-
on therapy with drug A is not the best next step, though either of these may be tried in parallel with
surgical evaluation.

2. A 32 yr old male with intractable seizures and a normal MRI scan undergoes a non Invasive
evaluation. Based on scalp EEG and seu:ure semiology, he had subdural electrodes placed and
seizures were localized to the left language-dommant supplementary motor area Following surgical
resectioo the patient will MOST likely exhibit :
A. Temporary paresis on the left.
B. Left inferior quadrantopsia
C. Temporary Mutism
D. Anosmia and Finger agnosia

Resections m the language-dominant SMA often result in post-operative muteness, paresis (on the
CONTRALATERAL side - in this case right), and neglect The paresis may be contralateral but can
affect bilateral extremities as well The hallmark of these deficits is U1at they are trans~enl They
largely resolve within 3-4 weeks of surgery. Therefore if seizures are severe and debilitating, surgical
resection may well be medicated after thoroughly counseling the patient about temporary post-
operative defiats. Anomia and finger agnosia are characteristic of Gerstmann's Syndrome, resulting
from dominant parietal damage. Inferior quadrantopsia Is characteristic of damage to the superior
component of the optic radiations. in the parietooccipital lobe.
3. Patients with subcortical band heterotopia are characterized by:
A. Male predominance.
B. Subependymal giant cell astrocytomas.
C. Infantile spasms.
D. X-linked migrational disorder
Subcortical band heterotopia is an X-linked migrational disorder that results in a second layer of
subcortical gray matter in females. In males, it produces diffuse lissencephaly. Infantile spasms are
characteristic of West Syndrome. Subependymal giant cell astrocytomas are characteristic of
Tuberous Sclerosis.
--
4. The MOST common clinical feature of medl al temporal lobe seizures is:
A. Visual aura.
B. Ictal Oral Automatism
C. Ictal bicycling movements
D. Ipsilateral dystonic posturing
Oral automatisms are a very common ictal behavior during medial temporal lobe seizures and may
consist of lip smacking, chewing, swallowing or tooth-grinding. Bicycling movement of the lower
extremities are more commonly seen In complex partial seizures that originate In the frontal lobe,
particularly supplementary motor area. Gustatory or olfactory aurus, not visual auras, are commonly
seen in medial temporal lobe seizures. Dystonic posturing, while common in mesial temporal seizures,
is reliably contralateral. Mesial temporal seizures may be historically related to febrile convulsions
most commonly found In early childhood in up to 67% of people with documented medial temporal
lobe epilepsy. Epigastric sensations are frequently reported as auras as well.
--
5. West’s syndrome is BEST characterized by which of the following seizure type :
A. Primary and secondarily generalized
B. Generalized tonic clonic
C. Complex partial
D. Infantile Spasm
E. Atonic
West’s syndrome produces seizures n early chilhood and consist of infantile spasm, mental
retardation, and hypsarrhytmia (complete disorganization) on EEG. With maturity however, these
patients can subsequently develop other seizure types. Classically, west’s syndrome is treated with
ACTH rather than antiepileptics. Vigatrabine is one AED with improved control of infantile spasps –
results compared to ACTH are mixed.
--
6. Which of the following structures is BEST described as lateral to the hippocampal complex
(hippocampus, subiculum and parahlppccampal gyrus)?
A. Brain stem.
B. Ambient cistern.
C. Posterior cerebral artery.
D. Fusiform gyrus
E. Occulomotor nerve
The fusiform gyrus Is lateal to the hippocampal complex. The fusiform gyrus has a functional role In
color processing, face and body recognition, word and number recognition, and within-category
linguistic processing.
The brain stem, oculomotor nerve, ambient cistern, and posterior cerebral artery are all In close
proximity and are medial to the hippocampal complex. The anatomy of these structures is best
described in relationship to the brainstem, specifically the midbrain. The oculomotor nerve exits the
brain stem in the interpeduncular cistern and therefore lies anterior and medial to the hippocampal
complex. The posterior cerebral arteries arise from the terminal division of the basilar artery above
the level of CNIII and can be visualized on the lateral surface of the midbrain as it traverses towards
and through the ambient cistern.

1. A 28 year-old healthy woman who delivered her second child 10 days ago presents with new onset
headache. lethargy. and confusion. She has no focal deficits. A non-contrast head CT Is obtained and
shown below. What is the most appropriate definitive treatment for this patient?
A Endovascular embolization
B Recombinant activated Factor VII
C Intravenous heparin infusion
D craniotomy for hematoma evacuation
E. Observation only
The scenano depicted above is indicative of intracerebral hemorrhage from cerebral sinus
thrombosis, making anticoagulation with continuous intravenous heparin infusion the most
appropriate answer. Small hernorrhages are evident on CT (white arrows). as well as a hyperdense
superior sagittal sinus. Likely representing sinus thrombosis. The diagnosis of sinus thrombosis is
reinforced by the clinical history of recent childbirth; cerebral sinus trombones is more likely in
patients with a hypercoagulate state, such as the peripartum period (within 16 days of childbirth).
Intracerebral hemorrhage often secondarily occurs in patients with cerebral sinus thrombosis due to
venous hypertension. Treatment goals for a patient with sinus thrombosis are prevention of further
thrombosis and recanalization of the occluded sinus. Initial treatment should be administration of
anticoagulation with a continuous intravenous heparin infusion.

--
2. a 47 year old woman presented with a frost seizure and underwent magnetic resonance imaging of
the brain with multi voxel spectroscopy. The spectroscopy shown in figure 1 is MOST consistent with
what diagnosis :
A. bacterial abcess
B. Glial Neoplasm
C. Toxoplasmosis
D. Demyelinating Plaque
E. Meningioma
Proton magnetic resonance (MR) spectroscopy is a well-established technique for studying the
chemical composition of structures. Hence, it provides a potential noninvasive method of analyzing
the metabolite content of normal and diseased brain. A typical MR spectroscopy plots the relative
concentration of a given metabolite versus the effect that the metabolite has on the rotational
frequency of protons within the sample (measured in parts per million [ppm]). Several standard peaks
are measured by MR spectroscopy of the brain in routine clinical J:lractice: N-acetylaspartate (NAA)
is a marker for neuronal viability, creatine is generally used as an internal marker or reference, and
choline is involved in the synthesis of phospholipids and is a marker of cellular turnover. The
presence of additional peaks can be a helpful differential feature in certain disease states: glutamine
and glutamate are astrocyte markers and may be altered in the setting of neuronal damage,
myoinositol is an astrocyte marker elevated in Alzheimer disease, and lipids and lactate are elevated
in inflammatoy processes and in necrotic tissue.
--

3. A 48 year-old man with a right middle cerebral artery distribution infarction. Despite aggressive
medical and endovascular therapy, the artery remains occluded. Two days later he has a decline in
mental status. His head CT is shown. What is the most likely explanation for the patient’s new
neurological
deterioration?
A. Occlusive Hyperemia
B. Reperfusion syndrome
C. Hemorrhagic conversion
D. Cerebral Edema
E. Normal perfusion pressure breakthrough

The most likely cause of acute deterioration 2 days after a completed large territory stroke is cerebral
edema. The CT demonstrates a large right MCA infarct associated with mass effect and midline shift
from cerebral edema. No sign of hemorrhage is present on the CT scan. The patient presented outside
the window for intravenous recombinant tissue plasminogen activator (tPA) or endovascular therapy.
Therefore, hemorrhagic conversion and reperfusion are unlikely causes of his mental status change
--

4. The appropriate maintenance rate for IV fluid in a 2 week old child is:
A. 4 ml/kg/hr
B. 10 ml/kg/hr
C. 2 ml/kg/hr
D. 20 ml/kg/hr
E. 1 ml/kg/hr
The appropriate maintenance IVF rate for a hospitalized infant from 3 days to 3 months old is 4 mll
kglhour. 2 ml/kg/hour may be appropriate on the first day of life, but not after three days of life. The
other rates are not appropriate for infants under standard circumstances.
--
5. What factor MOST indicates the need for enhanced screening for coronary ischemia prior to
general anesthesia for a neurosurgical procedure:
A. male sex
B. orthostatic intolerance
C. age > 60 years
D. surgical urgency
E. neoplastic disease

Orthostatic intolerance and other historical risk factors for coronary ischemia are important
indicators that a more thorough pre-operative workup for coronary ischemia should be completed,
often in consultation with a cardiologist. Screening studies may include, in addition to a 12 lead
electrocardiogram, echocardiography, stress or pharmacological-stress echocardiography, coronary
angiography or other imaging, etc.
When performing a medical history, the neurosurgeon should "seek to identify serious cardiac
conditions such as prior angina, recent or past myocardial infarction, congestive heart failure, and
symptomatic arrhythmias and also determine whether the patient has a prior history of pacemaker or
implantable

1. A patient presenting for resection of a small parietal meningioma has a preoperative INR of 1.5.
What is the likellhood that the transfusion of 2 units of FFP would normalize her INR?
A. 50%
B. 25%
C. <5%
D. 75%
E. 95%

Abdei-Wahab prospectively audited all FFP transfusions for an INR of 1.1-1.85 at Mamusetts
General Hospital over 13 months (324 transfus100s had the necessary follow up data). Transfusion of
FFP resulted in normalization of PT-INR values in 0.8% of patients and decreased the INR halfway
to noonalization in 15% of patients. Interestingly, !here was no significant relationship between
pretransfusion INR and likelihood of achieving 50 percent cooedion of the INR after FFP transfusion
There was no dose-respoose effect, and increasing amounts of FFP did not appear to result in larger
decrements in INR. Median decrease in INR was 0.07.
--
2. During the creation of a burr hole for a deep brain stimulator, an awake, sedated patient develops
the sudden onset of coughing, hypotension, and hypoxia. The clinical presentation suggest which of
the following complications:
A. Intracranial hemorrhage
B. Aspiration pneumonia
C. Air embolism
D. Tension pneumocephalus

During creation of the burr hole In awake patients, sudden vigorous coughing may be a sign of
venous air embolism. Other signs are unexplained hypoxia and hypotension. Early detection may be
possible with precordial Doppler monitoring. The overall incidence of venous air embolism as
detected by a precordial Doppler uTirasound has been reported to be4.5%. Hooper et al. in their
small study of 21 patients noted 1 venous air embolism (1 of 221ead Insertions), and the important
predictors were patient positioning and the occurrence of coughing.
--

3. In a Nationwide lnpatient Sample, the most common cause of perioperative visual loss (POVL) In
465,345 patients undergolng spinal fusion was:
A Anterior ischemic optic neuropathy
B Corneal abrasion
C Cortical blindness
D Posterior ischemic optic neuropathy
E. Retinal vascular occlusion
The most cause of POVL after spinal fusion is cortical blindness. The prevalence an spinal fusion is
3.09/10,000 cases. While the precise cause is unknown, risk factors include male gender, age less
than 18 years, anemia, and requirement for blood transfusion. Theoretical risk factors also include
prone position and intraoperative hypotension.
--
4. In a 2009 study of 5.6 million patients who underwent the principal procedures of knee
arthroplasty, cholecystostomy, hip/femur surgical treatment, spinal fusion, appendectomy, colorectal
resection, laminectomy without fusion, coronary artery bypass grafting, and cardiac valve procedures
from 1996 to 2005, cardiac and spinal fusion surgery had the highest rates of POVL. The national
estimate in cardiac surgery was 8.64/10000.
The American society of Anesthesiologists (ASA) class is designed to estimate what parameter:
A. Patient physical status
B. Age maximum of surgery
C. Tolerable blood loss
D. Surgical risk
E. Operative mortality
--
5. The use of Intensive Insulin therapy to reduce blood glucose In critically Ill patients with
aneurismal subarachnoid hemorrhage has teen associated with an increase In the Incidence of what
complication?
A. Myocardial ischemia and Infarction
B. Hypoglycemic episodes
C. All-cause in-hospit31 mortality
D. seizures
E. Surgical site infection
--
6. What is the BEST estimate of blood volume in a thirteen month old child weighing 10 kilograms?
A. 700 ml
B. 1000 ml
c. 800 ml
D 900ml
E. 600 ml
--
7. What is the maximal reduction in the cerebral metabolic requirement for oxygen (CMR02)
achievable exclusively through the use of high-dose barbiturates?
A. 50%
B. 25%
C.15%
D. 75%
E. 90%
--
8. What is the most concerning side effect or dexmedetomidine Infusion In ICU patients?
A. Inhibition or the locus ceruleus
B. Bradycardia
C Systolic hypertension
D. Respiratory depression
E. Intracranial hypertension
--
9. Which anesthetic agent is least likely to precipitate EEG evidence or seizure activity?
A Lidocaine
B. Isoflurane
C. Enflurane
D. Propofol
E. Sevoflurane

1. A lateral view or the cerebrum is shown in Figure 1. During a pterional exposure or the Sylvian
fissure you encounter significant brain swelling. In order to achieve rapid brain relaxation, the BEST
point to access the frontal horn or the lateral ventricle with a catheter would be at number.
A.5
B. I
C. 2
D. 3
E. 4

The neurosurgeon should have a see-through type awareness of the position of the deep structures in
relation to the cortical surface. The frontal horn is located deep to the inferior frontal gyrus (2), the
atrium is located deep to the supramarginal gyrus (3), and the temporal horn is located deep to the
middle temporal gyrus (6) The fomtal horn could also be cannulated through the middle frontal gyrus
(1) With a catheter oriented perpendicular to the cortical surface This area however would not be
typically exposed in a pterional approach.
2. A lateral view of the cortical surface bordering the right sylvian fissure that is commonly exposed
in a frontotemporal craniotomy IS shown in F'19ure 1. The lower ends of the preoen1ral (1) and post
central (2) are are in the exposure. The supramarginal gyrus is indicated by number :
A. 9
B. 5
C. 6
D. 8
E. 7

The surface of the inferior frontal gyrus is formed, from anterior to posterior. by the pars orbitalis
(6), pars triangularis (5). and pars operrularis (4). The precentral gyrus (1) is positioned juSI behind
the pars operrularis. The area below the lower apex of the pars triangular (4), .Which is commonly
retractcd upward, away from the sylvian fissure, is the widest point along the fissure and a suitable
point to begin opening the fissure. The postcentral gyrus (2) iS located just anterior to the
supramarginal gyrus (7), which wraps around the uptumed posterior end of the sylvian fissure. The
angular gyrus (8) is behind the supramarginal gyrus. The lower part of the pre- and postcentral gyri
are commonly exposed at the posterior-superior edge of a frototemparal craniotomy in the area
between the pars operrularis and the supramarginal gyrus. The transverse temporal gyri on the
operrular surface of temporal lobe produce prominences (9 and 10) along the lower margin of the
sylvian fissure at their lateral end. The most anterior of these is Heschi gyrus (9). A gyral bridge (3)
commonly connects the lower ends of the pre and post central gyri and separates the lower
3. A view from above Into the anterior part of the third ventricle is shown In Figure 1. The BEST
location for performing a third ventriculostomy is indicated by number:
A. 8
B. 5
C. 6
D. 9
E. 7

This view into the anterior part of the third ventricle exposes the columns of the fornix ( 1 ), inferior
to the foramen of Monro, and the anterior commissure (2), crossing the anterior wall of the third
ventricle in front of the columns of the fornix (1) and just above the lamina terminalis (3). The
chiasmatic recess (5) is located between the upper margin of the posterior edge of the optic chiasm
(6) and the lamina terminalis (3). The posterior edge of the optic chiasm (6) is located below the
chiasmatic recess. The infundibular recess (7) extends into the base of the pituitary stalk, and the
mamillary bodies (9) are located above the apex of the basilar artery. An endoscopic third
ventriculostomy would be completed at (8), where the floor of the third ventricle is thinnest.
--
4. During a retrosigmoid approach for a small vestibular schwannoma resection, the artery shown In
the figure(arrow) Is most at risk for injury. This vessel Is the:
A. Superior cerebellar artery.
B. Anterior Inferior Cerebelar artery
C. Posterior Inferior Cerebelar Artery
D. Vertebral artery.
E. Basilar Artery

The anterior inferior cerebellar artery (AICA) Is most at risk during resection of a vestibular
schwannoma regardless of approach. It can have a variable orientation in the CPA and meatus. In up
to 40% of the cases it may actually deep Inside the IAC thus making it vulnerable to Injury even via a
middle fossa approach.
--
5. Figure 1 shows a posteorlorvlew of the fourth ventrlde. Willett of the foUowlng numbered
structures represents the hypoglossal trigone area?
A. 3
B. 2
C. 1
D. 5
E. 4

The median sulcus (1) divides the floor longitudinally in the midline. Each halter the floor Is divided
longitudinally by an Irregular sulrus. the sulcus limitans. which deepens lateral to the facial
colliculus (2) and hypoglossal triangles (5) to form the superior (1) and Inferior foveae, respectively.
The inferior fovea is located lateral to the hypogossal nucleus (5). A darkened area of cells, the locus
ceruleus (8), is located at the rostral end of lhe sulcus limitans and medial to the superior cerebellar
peduncle (6). The hypoglossal (5) and vagal (3) nuclei and the area postrema (4) are stacked one
above the other in the lower part of the floor to give the configuration of a pen njb; thus, the area Is
referred to as the calamus scriptorius. The stria medullaris crosses the floor at the level of the lateral
recess.

6. The BEST description of the relationship between the oculomotor nerve (CN III), P1 segment of
the PCA and the SCA is that:
A. It has inconsistent relationship to the PCA and the SCA
B. It runs above PCA and the SCA
C. It runs between the PCA and the SCA
D. It runs below the PCA and the SCA
Once the oculomotor nerve exits the midbrain it reliably travels between the P1 segment of the PCA
and the SCA

7. Three days after clipping of a ruptured anterior communicating artery aneurysm, a patient develops
contralateral paresis of the arm and face. as well as dysarthria A cerebral angiogram reveals focal
vasospasm that eventually results in infarction of the corresponding vascular territory, confirmed by
CT. What vessel was most likely affected by vasospasm in this case?
A Recurrent artery of Heubner
B. Anterior choroidal artery
C. Distal anterior cerebral artery
D. M1 branch of the middle cerebral artery
E. Posterior Communicating Artery

The recurrent artery of Heubner (also known as the medial distal striate artery) arises just distal (and
less commonly proximal) to the anterior communicating artery. It supplies the head of the caudate.
anterior limb of the internal capsule, anterior putamen and globus pallid us, the septal nuclei, and the
inferior frontal lobe. It is termed & idguo;recurrent&rdquo; due to its course from its origin
backwards towards the ipsilateral Proximal A1 The artery is often at nsk of mjury during surgical
treatment of an anterior communicating arterY aneurysm and may develop subsequent vasospasm.
Compromise of blood flow in the recurrent artery of Heubner classically results in contralateral arm
and face weakness, and occasionally dysarthria. Bilateral injury results in akinetic mutism. The CT
image demonstrates infarction of the caudate head and internal capsule resulting from vasospasm of
the recurrent artery of Heubner.
8. What structure passes through the foramen demonstrated with the white arrow?
A. V2
B . V1
C. Vidian nerve.
D. VJ
E. Optic nerve.

V2 travels throu h the foramen rotundum, which is the structure identified in the coronal CT in the
figure. The foramen shq'wed immediately inferior and medially located inside the sphenoid bone is
the pterygoid canal, Which contams the vidian nerve. The superior orbital fissure (SOF) is seen above
the foramen rotundum and the optic canal s seen above the SOF.

9. Which of the following structures of the superior orbital fissure pass around the annulus of Zinn:
A. Nasodliary nerve.
B. Inferior branch of oculomotor nerve.
C. Superior branch of oculomotor nerve.
D. Abducens nerve.
E. Trochlear nerve.

The correct answer is the trochlear nerve. Four structures pass lateral to the annulus of Zinn in the
superior fissure. These structures are the lacrimal nerve, the ophthalmic vein. the frontal nerve and
the trochlear nerve.
Both inferior and superior branches of the oculomotor nerve pass through the annulus of Zinn as does
the abducens nerve and the nasociliary nerve.
A PGY-2 resident has worked 75 hours in the current week. The resident began his call shift
yesterday at 06:00. The resident had 2 days completely off work in the previous two weeks. During
the call day, the resident admits a pediatric patient with a rare brain tumor.
The next day, the resident hands off call responsibilities to another resident. However, he then
stays to assist with a rare pediatric brain tumor resection until noon. Which ACGME duty hour rule
applies?
A. Adequate rest between duty periods
B. Minimum days off
C. Maximum duty period length
D. In house call limitation
E. 80 hour work week
The ACGME limits continuous duty hours not to exceed 24 hours, with an allowable extension for
continuity of patient care. Justifications for extensions include continuity of care for a severely Ill or
unstable patient, Academic Importance of the events transpiring, or humanistic attention to the needs
of a patient or family.
--
2. A Type 2 error (false negative) in interpreting clinical trials occurs when:
A. Inadequate informed consent was obtained in over half the cases.
B. A treatment is found to be ineffective but the sample size is too small.
C. A positive result relies on an incorrect statistical test
D. The test is not randomized or controlled.
E. Gender makeup of the experimental and control populations are not equal.
--
3. A medical device company is sponsoring a presentation regarding a new implant they have
developed for treatment of spinal instability. The presentation is followed by a modest meal at a local
restaurant. At the conclusion of the meal, the company representative passes out gifts to the attending
physicians, as a "thank you" for taking time out of their personal lives to attend a work meeting.
Which of the following gifts would be acceptable under the AMA code of ethics and Advamed
guidelines?
A. Tickets to professional basketball game the following week
B. Textbook of spinal anatomy with company logo Imprinted on cover
C. $100 gift certificate to restaurant
D. Golf shirt with company logo displayed on sleeve
Any gifts accepted by physicians should primarily be directed towards allowing the physician to
provide better care to patients. Accordingly, textbooks, modest meals, and other gifts are allowed if
they primarily are geared towards the provision of education Cash payments and gifts unrelated to
the physicians work are not allowed
--
4. A physician chooses to utilize a questionnaire-based outcome measure in his/her practice The
questionnaire is administered to a core group of patients and then readministered to the same patients
within a few days. The purpose of this exercise is to confirm that the replies to the same questions by
the same patients do not change after a few days. A statistical analysts is completed to assess the
degree of similarity between the first and second replies In this setting. This type of analysis for an
outcome measure is a reflection of its:
A. Responsiveness
B. Integrity
C. Reliability
D. Sensitivity
E. Validity
--
5. An appropriate way for a neurosurgeon working in a complex team environment to mitigate stress
and fatigue would be to:
A. Not acknowledge stress and fatigue, therefore reducing its effect on performance.
B Recognize stress and fatigue, thereby increasing the use of threat and error management
strategies.
C. Rely on previous experience of chronic stress and fatigue during residency training to overcome
these factors
D. Deny the existence of stress and fatigue in order to avoid distraction from the task at hand.
--
6. Certain medical conditions are subject to mandatory physician reporting. You evaluate a recent
immigrant from Mexico complaining of occasional back pain. He has not been previously evaluated
by a physician in the United states. Which of the following diagnoses are required by law to report, if
the patient reveals it to you as part of their medical history?
A.inactive TB
B. Assault
C. Polio
D. Cysticercosis
E. AIDS
--
7. Company X offers surgeons $100 for each time they implant their patented deviced. Which federal
statue has been violated?
A. EMTALA
B. Anti-Kickback
C. Stark
D. HIPAA
E. This is not a violation
8. Dr. Carter is investing with a group of surgeons who want to open an ambulatory surgery center
(ASC). In order for this investment to pass Anti-Kickback statutes, what should Dr. Carter&rsquo;s
admitting status be at the proposed ASC?
A. Nonadmitting
B. Consult only
C. Active status
D. Emeritus
In order for Dr. Carter to fall within a safe-harbor of the federal anti-kickback law, he must be an
active member of the staff and operate at the proposed ASC. Non-surgeon Investors are not allowed
to participate in ASC because of the potential for such physicians to refer to surgeons who use
facilities that those non-surgeons own.

9. Dr. Jones designed a new instrument, obtained a patent, and sold the device to Company X. Dr.
Jones then became an employee of Company X and draws a fair market value annual salary for
marketing the instrument. Under what conditions can Dr. Jones give a presentation about this
Instrument at a CNS or MNS sanctioned meeting?
A. In a break-out session
B. Only if he is rot the first author
C. in poster form only
D. with disclosures on the first slide
E. Dr.Jones cannot give the presentation
--
10. Dr. Jones is presenting a paper she co-authored at a CNS/AANS CME sanctioned meeting. She
has disclosed a conflict of interest (COl) w1th Company Z prior to the meeting. Her presentation will
include information about products made by Company Z. In order for the paper to be presented, Dr.
Jones must:
A. Limit her comments to research data and results.
B. Disclose COI without limiting comments.
C. Eliminate her COI with Company Z within one month prior to the meeting.
D. Have the paper presented by a co-author without COL
Dr. Jones may still speak at this meeting provided her comments are limited to the data and results of
her research. Even if another co-author were to present the same data, because Dr. Jones was a co-
author. The disclosures and limitations would transfer to the co-author. COI is retrospective 12
months, thus eliminating COl one month prior to the meeting would be insufficient.
--
11. During performance of a lumbar discectomy, there is a delay in radiology availability. The spinal
surgeon elects to proceed with laminotomy and disc space exploration but finds no herniated disc
material. A subsequent intra-operative radiograph demonstrates that the neurosurgeon is operating at
the wrong level. The spinal surgeon adjusts the level of approach and successfully completes the
procedure. In this circumstance, of the following. the neurosurgeon should:
A. Consult risk management committee and ask them to disclose the error
B. Disclose the error to a quality assurance committee, but not to the patient
C. Request that the patient advocate disclosure the error to the patient
D. Not disclosure the error because the patient was not harmed
E. Disclose the error to the patient in a timely fashion
--
12. Federal health care insurance agencies including Medicaid and Medicare have established spesific
standards for medical coding, biling , and reimbursement. Under these systems, primary responsibility
for accurate coding and biling rests with :
A. The surgical practice manager
B. The surgical coding specialist
C. The hospital biling departement
D. The operative surgeon
E. The hospital compliance officer
--
13. In order to provide useful, high quality medical evidence, a clinical study should employ an
outcome measure that has the following characteristics :
A. published, peer review, and objective
B. Subjective, straightforward, and simple to employ
C. Reproducible, nonbiased, and comprehensive
D. Parametric, nonbiased, and comprehensive
E. Reliable, valid, and responsive
--
14. Steep gradients In authority can negatively impact patient safety when:
A. Fear or litigation results in physician migration out or a community.
B. Surgical instruments are incorrectly counted at the beginning of a case.
C. An incomplete medical record results in a missed drug allergy.
D. Overcrowded operating rooms create delays in scheduling.
E. A nurse who is unable to reach a resident and does not call an attending
--
15. The agencies for Healthcare Research and Quality (AHRQ) is charged in part with promoting
evidence-based medical practice. AHRO reports recommend standards, Guidelines and Options for
practice that are based on a scale of evidence class (or quality) from the peer-reviewed, clinical
literature. In the case of a retrospective. Case-controlled study of craniotomy plus whole brain
radiation therapy versus stereotactic radiosurgery plus whole brain radiation therapy for solitary lung
cancer brain metastasis. The evidence class is:
A. Not classifiable
B. Class III
C. Class I
D. Class II
--
16. The Joint Commission on the Accreditation of Health Care Organizations (JCAHO) has issued a
set of National Patient Safety Goals to be implemented by all hospitals and hospital-based
practitioners by January 1st, 2005. One of JCAHO's 2005 national patient safety goals is:
A. Include the use of the patient room number as a patient identifier whenever administering
medications or performing procedures.
B.Standardize a list of abbreviations, acronyms and symbols that are not to be used throughout
the organizatlon
C. Assure that the full range of manufactured drug concentrations and Intravenous solutions are
readily available within the hospital.
D. Implement a process to mark the surgical site involving nursesiO.R. technicians.
E. For verbal telephone orders, verify the order by having the person receiving the order read-back the
key portions.
--
17. The class of evidence associated with a controlled study of severe closed head injury
demonstrating reduced brain parenchymal loss and enhanced residual motor function in rodents
treated with hypertonic saline versus mannitol after injury is:
A. Class I
B. Class II
c. Class Ill
d. not classifiable
--
18. What is the ACGME definition of systems-based practice?
A. Make informed decisions about diagnostic and therapeutic interventions based on patient
information and preferences, up-to-date scientific evidence, and clinical judgment.
B. Demonstrate an investigatory and anal~ic thinking approach to clinical situations.
C. Know and apply the basic and clinically supportive sciences which are appropriate to their
discipline.
D. Partner with health care managers and providers to assess, coordinate. and improve health
care.
E. Demonstrate sensitivity and responsiveness to patients' culture, age, gender, and disabilities.
--
19. When looking at the total cost of medical malpractice in the U.S., what fraction of the total money
involved (including legal expenditures, settlements, and judgemenets) ends up going to the plaintiff
(patient)?
A. 1/5
B. 3/4
C. 112
D. 1/3
One criticism of the medical malpractice system in the U.S. is that it is very inefficient. While only 2%
of patients that are injured by a physician ever file a claim, only 17% of those who do file a claim
appear to have suffered from real physician negligence. All of this makes for a system that has high
transaction costs and has the majority (~62%) of the money going toward the payment of attorneys
with only ~38% going to plaintiffs.
--
20. Which of the following activities must be submitted to institutional review board (IRS) for
supervision?
A. You investigate the relationship between socioeconomics and post-concussion syndrome by
reviewing hospital records to correlate return visits with residential Zip Code.
B. In response co an elevated CSF shunt infection race. the hospital infection control service
prospectively observes 20 shunt operations and discover that holes in gloves correlate with infection
Double gloving is adopted
C. A school-age child suffers a gun shot wound to the brachial plexus and subsequently develops a
neuropathic pain syndrome. You prescribe an off label use of gabapentin.
D. An 11 year old harbors an incidentally discovered internal carotid aneurysm. You treat this lesion
with endovascular placement of coils approved by the FDA for use in patients 18 years old and older.
E. All neurosurgeon at your institution shave the scalp for cranial surgery. In response to an article
about shaveless neurosurgery, you adopt this practice.

1. A 27 years old pregnant woman presents with painless progressive vision loss in the right eye.
Examination reveals no light perception or direct papillary response in the right eye. A pale right optic
nerve, and a superior temporal visual field defect in the left eye. The etiology of her signs and
symptoms is most likely :
A. Pituitary apoplexy
B. Optic neuritis
C. PRES ( Posterior Reversible Encephalopathy Syndrome)
D. Suprasellar meningioma
E. Functional vision loss
The constellation of findings is most consistent with a supra sellar lesion. Meningiomas, pituitary
adenomas, and glial tumors are known to enlarge during pregnancy. Progesterone receptor positive
meningiomas as well as prolactinomas are known to enlarge during pregnancy causing chiasmal
compression. Pituitary tumors will typically present with bitemporal blur or vision loss confirmed on
formal visusal field testing. Visual acuities and color vision may be mildly affected, and pupil testing
is typically normal.
--
2. A 28 years old presents with sub acute onset of dropping of the eyelid and diplopia. He has a long
standing history of headaches but no history of trauma. Facial and eye photos are shown. Which is the
correct diagnosis?
A. Third nerve palsy
B. Sixth nerve palsy
C. Horner’s syndrome
D. Gaze palsy

This patient has a non – pupil sparing third nerve palsy, characterized by complete ptosis, anisocoria (left
pupil larger than right), and paralysis of left medial rectus muscle. Cavernosus Sinus region neoplasms,
posterior communicating artery and ischemia (atherosclerosis or diabetic etiology) are the most common
causes of third nerve palsy. Up to 20% of the cases may have an undetermained cause. Carotid dissection
is suspected in someone who presents with sudden onset of unilateral neck pain, facial pain, and
headache, with subsequent cerebral or retinal ischemia. Less commonly , patients may complain of
difficulty swallowing and the onset of pulsatile tinnitus. The most common finding on examination is a
partial Horner’s syndrome, paresis of hypoglossal nerve, and carotid bruit. A carotid dissection rarely
presents with an acute nerve palsy. Ischemic causes of a third nerve palsy spares the pupilomotor fibers
that reside on the surface of the third nerve. Compressive causes (trauma, tumor, and aneurysm) affect the
pupilomotor fibers in 95% of the time.
--
3. A 55 year old woman developed a severe headache and complains of acute vision loss due to
severe bilateral ptosis. Examination also reveals complete ophtalmoplegia of both eyes. This is MOST
consistent with which disorder :
A. Complicated migraine
B. Kearns – Syre syndrome
C. Myasthenic crisis
D. Pituitary apoplexy
E. PCOM aneurysm
This is an apopletic event and occurs with infarction and hemorrhage of the pituitary gland.
Depending on the size of the hemorrhage, this may involve the optic apparatus and cause vision loss
also. However, when the hemorrhage involves primarily the cavernosus sinus then complete
ophtalmoplegia and ptosis result. The rate of onset as well as presence of pain is inconsistent with
myasthenia. A posterior communicating artery anurysm would only cause ptosis with ophtalmoplegia
in one eye. A complicated migraine would not cause any occular motor problems except in a
migrainous third nerve palsy, which would bu unilateral again. The Kearns – Sayre syndrome is a
mitochondrial disorder that causes slowly progressive ptosis and ophthalmoplegia.
--
4. A patient complains of brief episodes of bilateral vision loss when she bends over or stands up
quickly, she also admits to pulsatile synchronous tinnitus. The MOST likely problem associated with
this is :
A. PCOM Aneurysm
B. Carotid – cavernous fitula
C. Pseudotumor cerebri
D. Internal carotid atherosclerosis
E. Occipital meningioma
Papiledema is associated with increased intracranial pressure from variety of etiliogies. Very
common complaints from patients include transient visual obscurationts and pulsatile tinitus. Patient
with carotid cavernous fistulae may also complain of pulsative tinnitus, but not transient visual
obscurations. Carotid atherosclerosis and not bilaterally and simultaneously. Occipital lobe
meningiomas and posterior communicating artery aneurysms do not cause these symptoms unless
somehow they have resulted in increased intracranial pressure that would cause papiledema to
develop.

1. A 50-year-old male banker present with several month history of a “ hissing steam” sound in the
left ear associated with occasional muffled hearing, he has had five episodes of profound vertigo that
were sudden in onset and have twice forced him to the ground for a 5 to 15 minutes. He experienced
nause and vomitting with most episodes and had to rest for several hours after each ‘attack’ magnetic
resonanceimaging of the brain is normal. An audiogram identifies low frequency hearing lost with
preserved speech discrimination. What is the most likely diagnosis :
A. Vestibular neuritis
B. Acoustic neuroma
C. Meniere’s disease
D. Benign positional vertigo
E. Vertebro-basilar insuffciency
Labyrinthine desease is the most common cause of true vertigo. Meniere’s disease, which is though to
be caused by a derangement of endolymphatic fluid regulation. Is the classic variety of labyrinthine
disfunction the clinical triad to meniere’s deseaseincludes intermitten severe episodes of disabling
vertigo. Tinnitus (often described as resembling the sound of escaping steam), and fluctuating flow
frequency hearing loss. Attacks may last several minutes and often several require several hours of
full recovery. Nausea, vomitting, and diaphoresis often acommpany the vertigo. A small portion of
patients with meniere’s desease experience sudden, violent falling attacks. Clinical onset is commonly
in middle age, and the disorder affect up to 1 per 100.000 population. The fluctuating hearing loss
mentioned is common and progress to deafness in the
--
2. A-61-years old airline mechanic presents with gradually worsening he hearing in his right ear over
the last 18 months. He has been uneble to use the telephone with that ear for 6 months. His audiogram
(same result for bone and air conduction) is shown in figure 1. Speech audiometry reveals
discrimination score of 88% in the left ear that 65 dB and 20% in the right ear at 100 dB. His hearing
function is the best describes as :
A. Autosomal recessive deafness
B. Retrocochlear sensoneural hearing loss
C. Occupational high frequency hearing loss
D. Normal
The patients described in this question suffers from two problems : 1) bilateral, high frequency,
hearing loss and 2) profound right retrocochlear sensoneural hearing loss. His threshold for tones
conducted by both bone and air is diminished, consistent with a sensorineural cause of his bilateral
high frequency hearing loss (due to his work around loud jet engines). Aging and prolonged exposure
to load noise are the most common causes of high frequency sensorineural hearing loss. These causes
typically result in symmetric loss of hearing with a characteristic loss in higher (paticulary 400 Hz)
frequencies. Because low to mid frequencies are often intact in these persons, speech recognition may
be preserved. Although prolonged exposure to noise might explain the audiological testing results in
the patients left ear, the profound loss of hearing on the right raises suspicion of an addtional cause
of unilateral hearing loss.
3. In the case of an intracanalicular vestibular scwannoma, what distinguishes the middle cranial fossa
approach from the retrosigmoid approach?
A. Decreased risk of injuring the greater superficial petrosal nerve.
B. Early of the identification of the facial nerve
C. Improved preservation of vestibular nerve function.
D. Decreased risk of CSF leak
Small intracanicular vestibular schwannomas in patients with preserved hearing can be approached
via suboccipital or middle fossa approach, the latter allows for early identification of CN VII because
of the nerves supero-anterior lacotion in the distal internal auditory canal. Importanly, however,
early identification does not necessarily make preservation of the nerve easier, especially if the tumor
arises from the inferior vestibular nerve. The greater superficial petrosal nerve (GSPN) may be
stretched during elevation of the temporal lobe during the middle fossa approach and thus damaged.
Preservation of vestibular nerve is not a distinguishing feature of the middle fossa over the retro
sigmoid approach. The CSF leak rate is similiar between the two.

1. 2.5 year old child presents with macrocephaly and poor height and weight gain. He has just begun
to walk, but has age appropriate language development. A head CT scan and MRI show a large pineal
region lesion. Serum beta HCG and AFP are negative. The MOST appropriate procedures is:
A. Endoscopic biobsy and CSF shunt placement
B. Cerebral angiogram
C. Stereotactic Biopsy
D. Open biopsy and possible resection

In this case, the pineal region lesion is not a timor, but a vein of galen malformation. Biopsy or
attempt at open resection would be disastrous. This case emphasizes the need to carefully iamge, and
in appropriate cases obtain angiography, for pediatric cases that may involve vascular lesions. Vein
of galen malformation present variably by age. Newborn children typically present in severe
congestive heart failure due to high flow shunting through the malformation, and often macrocephaly.
Older children may present first with hydrocephalus, usually with a slower flow lesion. Yasargil
classified vein of galen malformation in the following manner. 1 : pure fistula of ACA or PCA branch
to vein of galen, 2 : fistulous thalamoperforating vessels to vein of galen, 3 : combined lesion, and 4 :
plexiform AVMs. Treatment options include endovascular techniques (both transarterial and
transvenous), observation with management of hydrocephalus and rarely, open surgical techniques.
Prognosis is related to age at presentation, cardiac status and presence of calcifications in the basal
ganglia, as a marker of cerebral injury due to steal. In order children, non-operative management or
hydrocephalus management alone can be successful. In this case, angiography showed a slow flow
lesion which was seen to thrombose on flow
--
2. A 10 year old boy with a history of aqueductal stenosis and CFS shunt present with nausea,
vomitting and headache and a temperature of 100.5 he had a shunt revision for similiar symptoms 3
month ago. His brother has been sick lately too, with diarrhea and abdominal pain. His CT scan shows
a ventricular catheter in good position and normal ventricular size. A shunt series is norma. The
MOST appropriate diagnotic test is :
A. None
B. Shunt exploration
C. Shunt tap
D. ICP monitoring

Pasients presenting with sign and symptoms of shunt failure within the first few months after prior
shunt failure are at high risk of shunt infection. Studies indicate that recent shunt revision is a risk
factor for both shunt infection and mechanical shunt failure. Nevertheless, the symptoms of shunt
failure are sometimes non-spesific. Shunt infection occurs at a rate of up to 12% after shunt placement
and may present either as repeated shunt failure, or with overt symptoms and signs of infection. CSF
analysis is essential for diagnosis.
--
3. A 10 year-old girl presents with signs of accelerated pubertal stage, convergence-retraction
nystagmus, and impaired upward gaze. Which of the following lesions would likely be the cause of
the findings described?
A. Pontine glioma
B. Pineal region tumor
C. Acqueductal stenosis
D. Multiple sclerosis
Pineal region tumors occur in children of approximately 10 years of age. These lesions can result in a
dorsal midbrain syndrome, which includes a triad of vertical gaze palsy, pupillary light-near
dissociation and convergence retraction nystagmus. Convergence-retraction nystagmus is best seen
with a down going optokinetic drum. While congenital aqueductal stenosis cam present with gaze
palsies, it is not associated with pubertal acceleration and patiens most often an earlier age with
increased head circumference or headache
While multiple sclerosis (MS) could cause gaze palsies such as that described in this question, MS
usuallly present in the fourth decade of life, is much less common than pineal region tumors in this
age group, and is not associated with accelerated puberty. Pontine gliomas also often present with
diploplia due to abducens palsies. However, the ocular finding difer significantly from the dorsal
midbrain syndrome and pontine gliomas are not associated with pubertal acceleration.
--
4. A 10-year-old boy presents compalining of headache and imbalance. Examination discloses mild
pailedema, right-sided dysmetria, and taxia. Magnetic resonance imaging with gadolinium
enhancememnt is shownin figure 1. The MOST important prognostic factor this patient is :
A. The extent of resection of the enhancing mass
B. The extent of resecion of the cyst walls
C. The presence of endothelial proliferation of histology
D. The presence of mitoses on histology
E. The presence of hydrocephalus at presentation
Approximately one third of posterior fossa tumors in children are astrocytomas (with the other most
common diagnoses being medulloblastoma and ependymoma). The large majority of astrocytoma are
benign, and most occur in the cerebellum. The most common pathological diagnosis in juvenille
pilocytic astrocytoma (JPA), with less frequent occurence of anaplastic astrocytomas and
glioblastomas. The typical magnetic resonance imaging or computed tomographic appearence of JPA
is a brightly enhancing mural nodule within a non-enhancing cyst, although some tumors may be
mostly solid. Pathological features that carry a poor prognosis in other tumors, including frequent
mitoses and endothelial proliferation, are compatible with world health organization grade 1
designation and favorable prognosis in cases of JPA. Rosenthial fibers are common but can also be
seen in other pathological entities. Total incision of the enhancing nodule or mass in a cerebellar
astrocytoma often results in indefinite disease-free survival. Adjecent cyst walls typically do not
enhance and are not neoplastic. JPAs of similiar pathological
--

5. A 15 year old male presents with severe low back pain. The pain responds poorly to ..... but
responds well to aspirin. CT demonstrates a 1.5 cm dense lytic lesion of with a calcified nidus and
circumferential sclerosis. What is the diagnosis?
A. Osteochondroma
B. Osteoid osteoma
C. Hemangioma
D. Oteoblastoma

This child has an osteoid osteoma which typically presents between 10-20 years of age. The spine is
involved in 10% of cases. CT reveals dense sclerosis around a lytic lesion with a central calcified
nidus of osteoid and woven bone. They are usually <2cm. Large lesion should raise the suspicion of
osteoblastoma. Symptoms include scoliosis and pain that respondsto nonsteroidal anti-inflammatory
medications. Hemangiomas present a at peak age of 30-50 years with a female predominance. 75%
are spinal and usually present in the vertebral body. CT demonstrates a honeycomb pattern. Most are
asymptomatic and require no intervention.
--
6. A 15 year old patient with a history of prematurity and post-hemorrhagic hydrocephalus presents to
the emergeny department with an intermittent fever of no higher than 100.2 deg . F, anorexia, and
mild headache. His peripheral WBC is normal, at 9.3. The patient and his mother both deny the
possibility of VP shunt malfunction. He states that “this is totally different than the symptoms i had 8
months ago when my shunt was blocked”. The MOST likely diagnos is :
A. Gastroenteritis
B. Pseudotumor cerebri
C. Proximal shunt obstruction
D. Bowel perforation
E. Abdominal pseudocyst
Fastidious skin bacteria may cause chronic ventricular shunt infection, which often presents as a
shunt related abdominal pseudocyst. Presenting symptoms include intermittent low grade fever,
abdominal fullness,anorexia, nausea and vomitting, ang headache. Fever >38,5 degrees C, rigors,
meningismus, and peripheral leukocytosis are rare. Infection typically occurs weeks to months after
the most recent surgical accessor or tap of the shunt system, representing the source of bacterial
contamination. Ventricular shunt tap from a valve or reservoir may yield a normal or slightly
elevated cerebrospinal fluid white blood cell cont. Cultures may grow slowly (up to a week after
sampling) or may be negative in the case of some diffucult to culture organisms. Work-up include
imaging of the abdomen to identify a peritoneal pseudocyst around the distal shunt catheter, which
indicates the presence
--
7. A- 2-year-old child undergoes computed tomography for the preliminary complaint of
macrocephaly (figure 1). He is developmentally normal and has normal results of the neurological
examination without papilledema. Six months later, routine follow-up magnetic resonance imaging
shows enlargement of the left middle fossa abnormality (figure 2) and repeat tasting reveals mild
developmental delay and early papilledema. Which diagnosis is MOST likely:
A. Arachnoid cyst
B. Epidermoid
C. Abscess
D. astrocytoma

Extra-axial cyst in the central nervous system of children are most commonly arachonid cyst,
although dermoid, epidermoid, neureneric, and other cyst types may occur. True arachnoid cyst have
cerebrospinal fluid (CSF) imaging characteristics on computed tomographic scans and magnetic
resonance images (including diffusion weighted imaging). Arachonoid cyst in the intracranial space
are not commonly found in the middle fossa-sylvian fisure (appoximately 40%), posterior fossa
(approximately 25%),and other locations (suprasellar, quadrigerminal cistern, interhemispheric
fissure, cerebral convexity). Arachonid cyst may spontaneously enlarge, involute, or remain static.
Cyst hemorrhage and associated subdural hemorrhage also are seen. Indications for surgical
management are controversional but may include progressive macrocephaly, cyst enlargement,
hemorrhage, headache, raised intracranial pressure, seizure, or neurological deficit. Surgical options
include cyst peritoneal shunt placement. Craniotomy for cyst fenestration, and endoscopic cyst
fenetration unless the cyst is fenetrated into central CSF cisterns
--

8. A year old presents with headache and vomitting. CT and MR imaging reveals the presence of a 4th
ventricular tumor. A suboccipital craniotomy is performed for total resection of a medulloblastoma.
Resection is followed protocol based chemotherapy and total neuroaxis radiotherapy. With a boost to
the posterior fossa. Which endocrinological complication is the MOST likely after radiotherapy for a
posterior fossa tumor in childhood?
A. Diabetes insipidus
B. Cushing’s syndrome
C. Growth hormone deficiency
D. Addison’s syndrome
Central nervous system (CNS) radiotherapy in childhood poses particular risks of hipopituitarism, in
particular of short stature from growth hormone deficiency. Diabetes insipidus is uncommon.
Gigantism, caused by growth hormone excess in childhood, is not seen. Pituitary neoplasms caused
by previous radiotherapy for other diagnoses are extremely rare. Endocrine dysfunction, shortstature
and developmental delay caused by early childhood radiation generally preclude the use of cranial
radiotherapy in children less than 3 years old.
--
9. A 6 year old child with a traumatic brain injury is intubated and appears agitated. His CT scan
obtained within the last hour shows a left parietal contusion. Intracranial pressure monitoring via
ventriculostomy shows sustained pressure above 20 mmHg, despite bucking againts the ventilator
with minimal stimulation. His vital signs are within age appropriate limits and he apprears well
hydrated. What is the most appropriate pharmacological management of this patient?
A. Morphine and versed
B. Pentobarbital
C. Vecuronium
D. Propofol

Sedation and analgesia are important adjuncts to the management of intracranial hypertension after
taumatic brain injury. The correct answer in this case is morphine and versed. They facilitate
maintenance of mechanical ventilation, vascular access catheters and ventriculostomyor other ICP
monitors. Pain and agitation increase cerebral metabolism, thereby producing an increase in cerebral
blood flow, volume and intracranial pressure. The advantages of these agents must be weighed against
their adverse effects on blood pressure and the neurological examination. Paralytics and high dose
barbiturates, while an option, eliminate the neurological examination. Propofol, while an exellent
choice of rapidly reversible sedation in adult patients is contraindicated for long term sedation in the
ICU and has been linked to a progressive acutely fatal cardiomyophaty and acidosis in children.
Paralytics and high dose barbiturates eliminate the ability to perform a neurological examination, and
paralytics without sedation are contraindicated.
--

10 A 9-year-old male presented to his pediatrian with headaches and growth delay. Pathological
specimens of his tumor obtained at operation are shown in the photomicrographs. What is the MOST
likely diagnosis:
A. Colloid cyst
B. Craniopharyngioma
C. Pilocytic astrocytoma
D. Hypothalamic hamartoma
E. Pituitary adenoma

Craniopharyngiomas represent 1-2% of all intracranial neoplasms and about 10% of the tumors of
the sellar region. The current WHO classification identifies two variants: adamantinomatous (fig 1)
and papillary (fig 2). The adamantinomatous type tend to be mostly cystic lesions, while the papillary
craniopharyngioma is most commonly a solid lesion. Some tumors may contain variable proportions
of these two histologic patterns. In children, most tumors present with endocrinologic abnormalities,
such as growth retardation and diabetes insipidus, and/or visual loss. In adults, symptoms of
compressive effects including visual defects and hypopituitarism may be present. Mild
hyperprolactinemia due to stalk compression may also be present.
--
11. A term infant is born with a cranial malformation in the occipital region (fig 1). Which diagnosis
is MOST likely?
A. Dermal sinus tract
B. Chiari III malormation
C. Myelomeningocele
D. Anencephaly
E. Encephalocele

Congenital encephaloceles are calvarial and dural defects containing neural tissue and cerebrospinal
fluid (CSF). Occipital encephaloceles are most common in north America, whereas frontoethmoidal
(sincipital) encephaloceles are more common in southeast Asia. Congenital basal (transethmoidal,
transsphenoidal) encephaloceles are rare. Occipital encephaloceles show a 70% female
predominance. The extracranial position of some neural tissue generally results in relative
microcephaly. Repair involves the resection of gliotic and nonfunctional neural tissue and
dural/cranial repair. Hydrocephalus and/or CSF leak not uncommonly complicate repair, thus,
placement of a ventricular shunt is often necessary. Myelomeningocele refers to similiar dysraphic
defect containing CSF and neural tissue, but which are dorsally open and occur in the spine. Dermal
sinus tracts in the region ofthe inion present as tiny pits with one or a few dark, dyplastic hairs; they
are often missed on a causal physical examination. Chiari III malformation involves herniation of
posterior fossa content through a dysraphic foramen magnum and cervical level dysraphic defect.
Anencephaly is a severe open dysraphic defect of the entire telencephalon
--
12. An 8 year old girl present with years of chronic headaches that have worsened during the past 6
months and prgressive difficulty in school. She had previously been an ‘A student’ but her grades
have dropped to Cs, and she complains of problems with memory, attention, and coordination. She
has mild papilledema. You recommend endoscopic treatment of her obstructive triventricular
hydrocephalus rather than VP shunt placement. During endoscopic exploration of the ventricular
system, the MOST appropriate site for surgical fenestration is:
A. Septum pellucidum superior to the fornices
B. 3rd ventricle floor anterior to the infundibular recess
C. 3rd ventricle floor anteior to the mammillary bodies
D. Lamina terminalis above the suprachiasmatic recess
Triventricular hydrocephalus, dilation of the lateral and third ventricles, result from obstruction of
cerebrospinal fluid flow in the posterior third ventricle, most commonly resulting from conginetal
acqueductal stenosis, hamartomas of the rectal plate, or pineal region tumors. Macrocephalus,
headache, papilledema, visual loss, cognitive deterioration, and other signs of raised intracranial
pressure, including death, may result. The gold standard therapy for >40 years has been ventricular
shunting, most commonly to the peritoneal space. However, ventricular shunts are subject to
mechanical and infectious complications and may result in extra-axial fluid or blood collections due
to cortical mantle collapse, particularly in patients such as this one with severe ventriculomegaly.
Alternatively, endoscopic third ventriculostomy represents a potentially curative single surgical
intervention by which the posterior third ventricular blockage is bypassed into the ventral
subarachnoid space(prepontine cistern). The ventriculoscope is introduced via a pre-coronal.
--

13. Children of mothers with diabetes mellitus have an increased insidence of which spinal disorder?
A. Meningocele manque
B. Sacral agenesis
C. Spinal dysraphism
D. Thoracic hemivertebrae
E. Intraspinal lipomas

Sixteen percent of children with sacral agenesis or caudal regression are born to diabetic mothers
and 1% of diabetic mothers have children with these conditions. The overall incidence of spinal
dysraphism has decreased 26% in the US by folate supplementation of cereal making this as well as
meningocele manque&acute, and intraspinal lipomas incorrect choices. Active or passive exposure to
tobacco smoke has been linked to preterm delivery or low birth weight, another example of a
maternal activity or condition being linked to a neonatal condition. There is no data to support
neonatal thoracic hemivertebrae being increased in mothers with diabetes.
--
14. Current pediatric head injury guidelines suggest maintenance of the minimum cerebral perfusion
pressure above 40 to 65 mmHg, depending on :
A. Skull thickness
B. Patient age
C. Status of fontanelle
D. Core temperature
E. Time since injury
As in adult patients, the insidence of intracranial hypertension incerases with a declining GCS score.
Limited clinical studies have suggested that a sustained ICP of > 20 is associated with significantly
worse outcome than if less than 20. Intuitevely, for younger children and particulary infants, ICP
treatment thresholds of ,10 would seem to offer better outcomes, but no study has to this point has
demonstrated this. Cerebral pefusion pressure targets for children are imprecisely known. One study
has suggested that CPP of <40 mmHg is associated with a poor outcome, while expert opinion holds
that velues between 40-65 mmHg, dependent on age, are appropriate targets. An open fontanella does
not precludethe development of intracranial hypertension, nor obviate the utility of ICP monitoring.
Diffuse cerebral edema after trauma without intracranial hemorrhage is thought to be more common
in children than adults, and may be an indication for ICP monitoring even when no evert mass lesion
is present. ICP monitoring for children even with GCS scores of 9 or greater mey be indicated for
certain mass lesions or if the patient cannot be followed with serial neurological examinations
15. The incidence of myelomeningocele has decreased during the past decade. Before the 1980s, the
incidence was 1 to 2 per 1000 live births. The current incidence is 3 per 10.000 live births. One reason
for this decline may be the recommendation by physicians and obstetricians for women to supplement
their folic acid intake. The american college of obstetrics and gynecology recommends the following
supplementation guideline to all women from menarche to menopause:
A. 400 micrograms every day
B. 800 microgramsduring first trimester
C. 4 gram every day
D. 4 gram every week
E. 400 micrograms after positive pregnancy test
The incidence of infants born with spina bifida has been declining for 50 years in the most areas of
the word. The incidence in 1984 was 5.9 per 10.000 live births. The factors contributing to decline
include improved maternal nutrition, prenatal screening and diagnosis, selective pregnancy
termination, undetermined enviromental changes, and vitamin supplementation, aspecially folat. The
incidence decrease from east to west in the united states. It is higher among females and is much less
common among blacks and asians. It is increased with poverty and poor nutrition. The american
college of obsetrics and gynecology recommends 400 micrograms daily of folic acid supplementation
(in addition to a healthy diet) to all women capable of becoming pregnant. Because the full benefit of
folic acid is seen in mothers who begin supplementation at least 3 months before conception and
continue it throughout pregnancy.

1. A 37 year old man has a tonic – clonic seizure and is found to have an unruptured 2 cm AVM
located in the sensorimotor cortex. Angiography shows that there are no associated aneurysms and it
has both superficial and deep venous drainage. The patient alects to unergo radiosurgery for his
Spletzer – Martin Grade III AVM. The 3.6 cc AVM volume was treated with 20 Gy to the 50%
isodose line. Which of the following is the MOST likely clinical outcome :
A. Radiation induced deficit
B. Hemorrhage induced deficit
C. Complete AVM obliteration
D. Subtotal AVM obliteration
The most likely outcome is complete obliteration with no new deficits. The main factor that predicts
AVM obliteration after radiosurgery is radiation dose. The chance of obliteration is approximately
90%, 80%, and 70% for AVM margin doses of 20 Gy, 18 Gy, and 16 Gy, respectively. The chance of
radiation related complications is related to the AVM location, AVM volume, and radiation dose. For
larger AVM volumes, the radiation dose is typically decreased to keep the chance of radiation
relatedcomplications less than 5%. The primary disadvantage of AVM radiosurgery compared to
surgical resection is that patients continue to have a hemorrhage risk until the AVM is completely
obliterated. The latency interval after radiosurgey until AVM obliteration is typically between one
and for years. Although deficit related to hemorrhage is possible in this case, it is likely in patient
who presented with seizures and does not harbor peri-nidal aneurysms.

2. A 45 year old woman with medically refractory trigeminal neurlgia chooses raiosurgical therapy.
Which of the following is the MOST appropriate maximum radiation dose for trigeminal neuralgia
radiosurgery :
A. 20 Gy
B. 40 Gy
C. 60 Gy
D. 80 Gy
E. 100 Gy
The appropriate maximum dose for trigeminal neuralgia radiosurgery is 80 Gy. Although some of the
first radiosurgical procedures were for trigeminal neuralgia, the inability to clearly image the
trigeminal system limited the usefulness of this technique. However, with the advent of MRI,
physicians are able to visualize the trigeminal root for radiosurgery dose planning. A prospective ,
multi-institusional dose escalation trial showed that patients receiving a maximum radiation dose of
70 Gy or more had a significantly greater chance of complete pain relief compared to patients
receiving less than 70 gy. Later studies showed that the chance of bothersome facial numbness was
significantly higher for patients treated at doses above 90 Gy. Consequently, most radiosurgical
centers limit the dose for trigeminal neuralgia radiosurgery to 80 gy in the hope of achieving better
facial pain outcomes than reported with 70 Gy with an acceptable incidence of new facial sensory
loss. Other factors that have been associated with improved facial pain outcomes are normal pre-
operative facial sensation, increased dose to the brainstem and absence of prior surgery. Negative
predictors for pain relief after radiosurgery has included patients with trigeminal neuralgia related to
multiple sclerosis or patients with a constant pain component.
--

3. A 56 year old man with history of renal cell carcinoma presents with localized back pain but
without myelopathy. Radiographs showed normal alignment and no evidence of instability. MRI
showed a dorsal, enhancing paraspinal mass with extension into the spinal canal. Which of the
following is the MOST likely outcome of stereotactic radiosurgery for the spine metastasis :
A. Tumor stabilization but persistent back pain
B. Tumor stabilization with an improvement in back pain
C. Tumor stabilization with new onset myelopathy
D. Tumor progression causing spinal cord compression
E. Tumor progression causing vertebral collapse
The most likely outcome after stereotactic radiosurgery of this spine metastasis is tumor stabilization
with an improvement in back pain. Utilizing the general concepts developed over 30 years of
intracranial radiosurgery, it has become possible to perform high – dose, single fraction
(radiosurgery) radiation delivery to spinal and para – spinal lesions. Although the follow up
available after spine radiosurgery is limited, spine radiosurgery has been shown to be feasible, safe,
andeffective. Tumor control is achieved in the majority of patients, and improvement in axial and
radicular symptoms has been noted in more 90% of affected patients. Contraindications to spine
radiosurgery include mechanical instability. The chance of radiation – induced myelopathy or
vertebral necrosis after spine radiosurgery is very low.
--
4. A 59 year old man presents with high frequency sensorineural hearing loss, near normal speech
discrimination (<90% at 40 dB), and an MRI showing a uniformly enhancing 10 mm mass extending
into the internal auditory canal. The MOST likely complication of stereotactic radiosurgery of this
lesion is :
A. Diplopia
B. Facial numbness
C. Hearing loss
D. Facial weakness
E. Swallowing difficulty
The most likely complication after vestibular schwannoma radiosurgery is hearing loss. As a less
invasive alternative to surgical resection, stereotactic radiosurgery has been utilized increasingly
over the past 20 years for patients with vestibular schwannomas. Preservation of useful hearing
(speech discrimination scores greater than 50%) is possible in approximately 60% of patients after
radosurgery. Facial weakness or numbness occurs in less than 3% of patients. Diplopia or
swallowing difficulty is rare in large radiosurgical series, the need for later tumor resection has been
less than 5%
--

5. A 63 year old man with a history of non small cell lung cancer presents with headaches. An MRI of
the brain demonstrates 2 enhancing lesions each measuring 2 cm, with mild vasogenic edema and no
mass effect. Which of the following is supported by class 1 evidence regarding stereotactic
radiosurgery (SRS) and whole brain radiation therapy (WBRT)?
A. For patients with one to three metastases, SRS alone results in better early outcomes than
SRS plus WBRT
B. For patients with one to four metastases, SRS alone is associated with poorer survival than SRS
plus WBRT
C. For patients with one to three metastases. WBRT plus SRS boost prolongs survival as compared to
WBRT alone.
D. For patients with one brain metastasis. SRS provides improved survival compared to WBRT.
SRS alone results in better early cognitive outcomes compared to SRS + WBRT for patients with 1-3
metastases. Recent class I evidence reported by Chang et al (2009) demonstrated that early
neurocognitive outcomes are signiticantly better in patients with one to three metastases who received
SRS only, when compared to SRS plus up-front WBRT. This single institution study specifically
examined cognition using the Hopkins Verbal Learning Test; differences in cognition were found
between groups at 4 and 6 months. There was a high rate of salvage therapy requirement (87%) in
the SRS group, including surgical resection and WBRT.
--
6. A 65-year old female with a growing left intracanalicular 6 mm tumor and progressively worsening
yet still serviceable hearing presents with the MRI shown. What is the most appropriate management
strategy?
A. Surgical removal via translabyrinthine approach
B. Stereotactic radiosurgery
C. Surgical removal via retrosigmoid approach
D. Reassurance, observation, and follow-up MRI in one year
Stereotactic radiosurgery affords a very low chance of facial neuropathy and a reasonable chance of
hearing preservation. Hearing preservation may be more likely in patients for whom a lower dose is
delivered to the cochlea. This is usually possible when the tumor is smaller in volume.
Resection via a translabryinthine approach will not preserve the patient's hearing. Given the patient's
age, radiosurgery is usually preferable to open surgery via a retrosigmoid approach.

--

7. Which of the following cranial nerves is most susceptible to radiation injury after radiosurgery of a
cavernous sinus lesion'?
A. Trochlear
B. Abducens.
C. Oculomotor.
D. Trigeminal.
E. Optic.
Stereotactic radiosurgery affords a very low chance of facial neuropathy and a reasonable chance of
hearing preservation. Hearing preservation may be more likely in patients for whom a lower dose is
delivered to the cochlea. This is usually possible when the tumor is smaller in volume. Resection via a
translabryinthine approach will not preserve the patient's hearing. Given the patient's age,
radiosurgery is usually preferable to open surgery via a retrosigmoid approach.

7. Which of the following cranial nerves is most susceptible to radiation injury after radiosurgery of a
cavernous sinus lesion'?
A. Trochlear
B. Abducens.
C. Oculomotor.
D. Trigeminal.
E. Optic.
The cranial nerve most susceptible to radiation injury after radiosurgery of a cavernous sinus lesion
is the optic nerve. Although it appears that cranial nerves will tolerate high doses of radiation (60-70
Gy) when it is delivered in multiple fractions, cranial nerves are more susceptible to injury after
radiosurgery. Moreover, different types of cranial nerves appear to have distinct tolerances for
radiation. The special somatic sensory nerves (optic, vestibulo-cochlear) are the most susceptible to
injury after high-dose, single-fraction radiation (radiosurgeryj. Recent studies have shown that the
chance of visual loss is less than two percent when the optic nerves and chiasm receive doses less
than 12 Gy. General somatic nerves (oculomotor, trochlear, abducens, hypoglossal) are rarely
affected by similar doses. The afferent component of the trigeminal nerve (general somatic afferent) is
intermediate in radiation sensitivity. The length of cranial nerve that is irradiated and a prior history
of radiation exposure are also important factors related to cranial nerve injury after radiosurgery.

1. A 15 year old boy presents with neck pain. He is neurologically intact. A CT of the cervical spine
reveals an osteolytic lesion with multiple fluid-filled cavities involving the body of C4. What is the
most likely diagnosis?
A. Ewings sarcoma
B. Aneurysmal bone cyst
C. Osteosarcoma
D. Eosinophilic granuloma
E. Fibrous dysplasia
The most likely diagnosis is an aneurysmal bone cyst (ABC), ABCs present commonly in children and
radiographically appear as an osteolytic lesion with multiple fluid-filled cysts. They are considered
benign tumors. Primary malignant tumors of the bone indude osteosarcoma and Ewings sarcoma.
Osteosarcoma have both a lytic and blastic component on xrays and are typically referred to having a
‘sunburst appearance’. Ewing’s sarcoma is the second most common primary bone tumor in children.
Radiographically, they demonstrate diffuse destruction of bone and are associated with a periosteal
reaction which produces an “onionskin” appearance on xray.
--
2. A 19 year old male presents with tussive headaches located at me posterior base of the skull.
NeuroIogic examination reveals weakness of the hands bilaterally with hypesthesia. MR of the brain
and cervical spine are shown in the figures. What Is the BEST initial management strategy for this
presentation?
A. Posterior cervical decompression
B. Posterior fossa decompression
C. Syringo-subarachnoid shunt
D. Ventriculoperitoneal shunt
E. Anterior transoral odontoid resection

Suboccipital or posterior fossa decompression has long been used as part of the surgical treatment
for synngomyelia related to Chiari I malformation. In approximately 50% of patients with Chiari I
malformation and up to 90% of those with spinal cord symptoms an associated syrinx is present.
Posterior fossa decompression in patients with Chiari I malformation and syringomyelia is an
effective and safe initial treatment. Anterior transoral odontoid resection, placement of a
syringosubarachnoid shunt and posterior cervical decompression are typically reserved for patients if
further abnormalities of the craniocervical junction exist or standard decompressive techniques fail
or cannot be applied. Ventriculoperitoneal shunt placement is
--
3. A 26 year old male presents after motor vehicle crash with absent right and partially preserved left
lower extremity motor function (more than half of left leg muscles have less than antigravity
strength). Sensation to pain and temperature Is markedly diminished In the left leg. Proprioception is
markedly diminished in the right leg.
Neuro-imaging studies are obtained and depicted in Figures 1 and 2. Which spinal cord syndrome
BEST describes the injury:
A. Cauda equina
B. Brown-Sequard
C. Anterior spinal
D. Central cord

The spinal cord injury described would be best described as a Brown-Sequard syndrome. Classically,
the history and physical examination reveal ipsilateral loss of proprioception and vibratory sensation,
ipsilateral motor paralysis, and contralateral loss of pain and temperature sensation inferior to the
lesion. This pattern reflects a functional hemisection of the spinal cord. The central cord syndrome is
characterized by disproportionately greater motor deficit in the upper extremities than in the legs.
Buming upper extremity dysesthesias are also common in central cord injuries. Central cord injuries
are commonly seen with forced nedc extension in the presence of underlying cervical stenosis.
Anterior cord injuries are associated with ventral compressive pathology and have historically been
attributed to ischemia in the anterior spinal artery

4. A 27 year old sustains a Type II odontoid fracture. He is complaining of upper cervical pain, but his
neurologic exam is normal. Which of the following factors would increase his risk for non-union?
A. Age less than 30 years
B. Comminution of the dens
C. Anterior displacement of 3 mm
D. Basilar Skull fracture
E. Vertebral artery injury
Odontoid fractures represent approximately 15% of all cervical spine fractures. A type 2 odontoid
fracture is through the base of the neck of the odontoid, and is usually considered unstable. The
treatment options indude rigid immobilization or surgical stabilization/fusion. The factors predicting
nonunion with immobilization alone are displacement of more than 4 to 6 mm, advancing age
(possibly >50 years old), posteriorly displaced fractures, and patients with neurologic deficits.
Surgical treatment is indicated if displacement is >6mm, previous nonunion, or instability of the
fracture site while in rigid immobilization.
--
5. A 28 year old male presents to the Emergency Department with complaints of severe low back and
right leg pain after injury while working as a carpenter the previous day. He denies weakness,
numbness and gait or bowel/bladder disturbance. Examination reveals trace weakness of dorsi-flexion
on the right associated with pain. The BEST Initial management strategy for this patient is:
A. Epidural steroid injection
B. Seven day course of bed rest
C. Lumbar discectomy
D. Nonsteroidal anti-inflammatory medications
E. Posterior lumbar interbody fusion
This patient presents with acute low back pain and sciatica without a significant neurological deficit.
There are no ‘red flags’ to indicate the need for imaging. The vast majority of patients with acute low
back pain and sciatica will recover spontaneously over a four to six week time course. In the absence
of a clinically significant neurological deficit, it is usually inappropriate to perform any type of
surgery. Similarly, the use of invasive non-operative measures is also not advised due to their
inherent expense, risk, and lack of proven efficacy in this patient population. Bedrest results in
deconditioning and is not generally recommended. When used, bedrest should not be prescribed for
longer than four days. The use of over the counter pain medications is recommended as a treatment
modality for patients with acute low back pain and radiculopathy.
--

6. A 33 year-old man with known metastatic breast carcinoma presents with a one day history of
paraparesis and bladder incontinence. Her lower extremity motor strength is 2/5 in all groups. An
MRI of her thoracic spine reveals an isolated dorsal metastasis with cord compression at T9-T1O and
preserved alignment. After administering steroids, the next step in her course of treatment should be:
A. Single session spinal radiosurgery
B. Spinal radiation therapy
C. T9-1O spondytectomny with instrumentation
D. No additional treatment
E. T9-10 laminectomy
The most appropriate answer is urgent T9-T1O laminectomies and decompression of the spinal cord.
This patient has suffered rapid neurological deterioration including bladder incontinence. Steroids
alone do not offer definitive care for this problem. A randomized, controlled study by Patchell and
colleagues reported improved outcomes in selected patients treated with surgical decompression
compared to patients who were treated with radiation alone. Although spinal radiation is considered
one of the primary treatments for metastatic epidural spinal cord compression, in this case (in which
the patient presents with acute neurologic deficits), surgical decompression should be considered
first. Spondyleclomy is unecessary in a patient with isolated dorsal compression of the spinal cord.

--
7. A 35 year old woman presents with progressive upper and lower extremity myelopathy. MR
imaging reveals a well defined, contrast enhancing lesion 2 cm in diameter in her cervical spinal cord.
What is the most appropriate treatment for this patient?
A. Open biopsy of the lesion followed by appropriate chemotherapy and radiotherapy
B. CT guided biopsy followed by appropriate chemotherapy and radiotherapy
C. Serial MR imaging every three months
D. Complete surgical excision of the lesion, if possible

The most common histologies for intramedullary spinal cord tumors in this age group are
ependymomas and astrocytomas. Surgical intervention is warranted in patients with dinicaily
progressive intramedullary spinal cord tumors. Preoperative neurologic status is the single most
predictor of postoperative neurologic function
--

8. A 35-year-old man presents with a several week history of left calf weakness and urinary
retention. The MRI scan of the lumbar spine is most consistent with a myxopapillary ependymoma of
the conus medullaris. What is the most appropriate initial management of this patient?
A. Laminectomy and tumor resection
B. Laminectomy and biopsy
C. CT-guided biopsy
D. Spinal radiosurgery
E. Clinical observation with early repeat imaging
The most appropriate initial management of this patient’s probable ependymorna is laminectomy and
tumor resection. Positive prognostic factors include symptoms of less than one year. confinement of
the lesion to the filum terminale without infiltration or adherence to roots of the cauda equina and
gross total resection. Either laminectomy with biopsy or CT guided biopsy would not be the most
appropriate treatment options as resection is the preferred approach. Radiosurgery has not been
proven effective as a primary therapy for spinal ependymomas
--

9. A 36 y/o restrained driver presented after a MVC. He complained of leg numbness and weakness.
Examination revealed him to have 4-/5 strength in his proximal and 3/5 strength in his distal lower
extremities and an L1 sensory level. Lumbar spine radiographs and axial CT through the level of
Injury are shown in the figures. What is the BEST definitive management strategy for this type of
Injury:
A. TLSO brace
B. Kyphoplasty
C. Posterior spinal fusion
D. Anterior spinal fusion
The answer is posterior spinal fusion with distraction instrumentation followed by an anterior
procedure if adequate canal clearance is not obtained. The management of flexion distraction injuries
in older adults usually requires surgical intervention. Stand-alone short segment constructs have been
demonstrated to have a high failure rate in the presence of significant deformity of the anterior and
middle elements and therefore a longer segment posterior fusion is indicated. While an anterior
approach may provide useful supplementation to a posterior approach the management of fiexion
distraction injuries, it does not directly address the disruption of the posterior ligamentous complex
seen with this type of injury. The results of bracing in this population are usually unpredictable as the
injury is also ligamentous. Kyphoplasty is useful for painful compression fractures but has no role in
the management of flexion distraction Injuries.

10. A 38 year old man presents with severe left leg pain. The pain came on spontaneously and he
cannot remember any antecedent trauma or illness. The pain is severe and has prevented him from
working as a carpenter since its onset two weeks ago. His examination Is notable for a positive
straight leg raising test on the left and paresthesias in the S1 distribution on the left. The remainder of
his physical and neurologal examination is normal. An MRI was ordered by his primary care
physician, and it reveals a left sided herniated disc. The next appropriate management strategy for this
patient is:
A. Posterior lumbar interbody fusion
B. Minimally incisional endoscopic discectomy
C. Analgesics and Physical Therapy
D. Percutaneous Chymopapain injection
E. Traditional open discectomy

Because of the absence of dinical “red flags”, the patient is not a candidate for any surgical
procedure in the absence of significant neurological deflcits unless his symptoms persist for at least
six weeks. Conservative (i.e. non-operative) management should be employed at this point in time.

11. A 42-year-old woman is referred with biopsy proven chordoma within the L3 vertebral body. MRI
does not show any ventral epidural extension or involvement of the peddes or posterior elements, The
BEST management option for this patient Is:
A. L3 spondylectomy and L2-4 fusion
B. DebuIking of tumor and L2-4 fusion
C. L3 corpectomy and fusion plus chemotherapy
D. Radiation therapy alone
E. L3 corpectorny and fusion plus radiation therapy
Chordomas are the most common primary malignant tumor of the spine. Although considered not to
possess significant metastatic potential, such lesions are locally aggressive, leading to neurologic
compromise and lytic destruction of bone. En bloc resection has afforded patients the greatest chance
of local control and disease-free survival. A spondylectomy is the most widely accepted approach,
consisting of resection of the posterior elements, pedicles and vertebral body. Such radical resections
may be assooated with significant surgical morbidity. Chordomas are generally considered to be
resistant to radiation therapy and chemotherapy. However, recent advances in photon and proton
radiation therapy and use of monoclonal antibodies may provide improved outcomes for poor
surgical candidates and for tumors that recur after surgery
--
12. A 43 year old man with a 3 month history of numbness and tingling in bilateral lower extremities,
diffioulty ambulating, and weakness in both hands and arms undergoes MR imaging of his cervical
spine revealing a homogeneously enhancing intramedullary spinal cord tumor. The tumor diffusely
and symmetrically enlarges the involved portion of the spinal cord without cyst formation. At surgery,
intraoperative frozen section yields ependymoma. The MOST appropriate treatment strategy is:
A. Attempted gross total resection
B. Biopsy and external beam radiation
C. Tumor debulking and duraplasty
D. Subtotal resection and radiotherapy
In cases of intramedullary ependymoma, a plane usually occurs between the spinal cord and tumor
These tumors tend to expand symmetrically within the spinal cord and may be approached via a
dorsal midline myelotomy. Spinal ependymomas are often reddish brown, enhancing the surgeon’s
ability to distinguish normal from neoplastic tissue. They may be totally excised in the majority of
cases. often with limited neurological morbidity. Therefore, debulking the tumor and/or referring the
patient for radiotherapy are less appropriate choices. After gross total resection, patients should not
undergo postoperative radiotherapy but should instead be followed with serial imaging. However, it
may be appropriate to administer radiotherapy to those patients where total resection was not
possible. The best predictor of functional outcome after spinal ependymoma resection is the patient’s
preoperative neurologic status.

13. A 45 year old gentleman is seen in the PACU after undergoing an anterior cervical discectomy
and fusion with plating at C6-7. He is noted to have a constricted pupil and ptosis. What is the most
likely explanation of this condition?
A. Recurrent laryngeal nerve injury
B. Over distraction of the disc space
C. Vertebral artery injury
D. Stretch injury of the sympathetic trunk
E. Myastenia gravis exacerbation
The most likely explanation is a stretch injury of the sympathetic trunk causing a Horner’s syndrome.
Hornets syndrome results from an Interruption of the sympathetic nerve supply to the eye, and Is
characlenzed by the thad of miosis (constrked pupil), partial plosis, and loss of hemifacial sweating
(anhidrosis). In the anterior cervical discectorny approach. lateral exposure of the longus cdi musdes
can lead to exposure and injury of the sympathetic trunk. whh lies just anterior. This ocoirs in
approximately 1% of patients undergoing this approach, and usually recovers spontaneously. The
sympathetic trunk Is more commonly injured in the r cervical spine compared to the upper levels.
--

14. A 48 y/o female with a history of breast caronoma presents with severe neck pain for several days.
The neck pain resolves with recumbency and worsens with movement. Her neurological examination
is normal except for symmetric hyper-reflexia in all four extremities. Her primary disease Is well-
controlled and she is otherwise healthy. MR imaging is shown below (Figure 1) In addition to
appropriate adjuvant chemotherapy and radiation, the BEST treatment option is:
A. Occipito-cervical fusion
B. Posterior C1-2 transarticular screw fixation
C. transoral resection and posterior fusion
D. External orthosis only
The best option Is ocopito-cervical fusion, radiation therapy, and chemotherapy. The patient is a 48
y/o, otherwise healthy female a C1-2 metastasis. The patients MRI demonstrates involvement of C1,
C2, and the superior endplate of C3. There is no canal compromise. Indications for surgical
intervention In patients with spinal metastases include 1) the need to establish a diagnosis, 2)
treatment of spinal instability, 3) decompression of the spinal canal in patients with radioresistant
tumors or tumors that have progressed despite XRT, and 4) the potential for cure in patients with a
single small metastasis within the vertebral body.

15. A 48 year old men presents with progressive complaints of hand paresthesias, loss of fine motor
control, gait instability, and urinary urgency (MR imaging is shown below). She undergoes a posterior
decompression via an open-door laminoplasty from C3 to C7. On post-operative day number 3, she
complains of bilateral shoulder pain and demonstrates mild weakness of her right deltoid. The MOST
appropflate next step in the management of this patient is:
A. Bilateral upper extremity EMG
B. Bilateral selective nerve root injections
C. Emergent surgery for laminectomy/foraminotomy
D. Administration of corticosteroids
E. Orthopedic/Physiatry consult for “frozen shoulder”

The most appropriate initial step in the management of this patient is the administration of
cortIcosteroids. Dissociated motor loss (DML) is a well known. but infrequent complication of
extensive decompressive cervical procedures classically characterized by specific motor weakness of
the C5 myotome, shoulder pain, and no dermatomal sensory loss. The reported incidence ranges from
2% to 20%. Although the etiology is unknown, nerve root tethering and traction injury after an acute
anatomic shift of the spinal cord following the decompression is one hypothesis. Traditional
management consists of conservative therapies including cervical immobilization, steroids, and
analgesics Improvement typically occurs over many months.
16. A 50 year ol male presents with a 2 week history of low back pain radiating down the postero-
lateral thigh, lateral shin, dorsum of the foot and great toe on the right side. His neurologic exam is
intact except for a (+) right straight leg raise at 30 degrees and 4+/5 EHL strength on the right. The
most appropriate initial step in the management of this patient is:
A. Intradiscal Electrothermy (IDET)
B. Epidural steroid injection
C. Microdiscectomy/foraminotomy
D. EMG/Nerve conduction studies
E. Education/Counseling
The majority of patients with pain secondary to radiculopathy used by a herniated nucleus pulposus
recover spontaneously without surgery. The initial step in treating such patients is educating them on
the natural history of the disease and reassuring them that the majority of patients do well without the
need for surgery. Conservative measures should then be applied for a period of several weeks or
longer, if at all possible. Conservative treatments include short periods of rest, anti-inflammatory
drugs (NSAID5), opioids, muscle relaxants, physical therapy, injections, and other modalities.
Patients suffering for more than 60 days from disc herniation have a statistically worse outcome than
patients suffering for less than 60 days.
--
17. A 50 year old male presents with a 2 week history of low back pain radiating down the postero-
lateral thigh, lateral shin, dorsum of the foot and great toe on the right side. His neurologic exam is
intact except for a (+) right straight leg raise at 30 degrees and 4-/5 EHL strength on the right. The
most likely diagnosis is:
A. L4-L5 far lateral disc herniation
B. L5-S1 foraminal disc herniation
C. L3-L4 paracentral disc herniation
D. L3-4 central disc herniatlon
E. L5-S1 paracentral disc herniation
This patient demonstrates a right L5 radiculopathy. Of those listed, the most likely lesion to cause a
right L5 radiculopathy is a right L5-S1 foraminal disc herniation. The L5 nerve exits its respective
neuroforamen at L5/S1. The classic paracentral disc hemiation at this level would have resulted in an
S1 radiculopathy. Far lateral and foraminal disc herniations are most likely to affect the exiting nerve
root at that level while central and paracentral herniations most likely affect the traversing nerve
root.
--

18. A 54 year old male with non-small cell lung cancer (NSCLC) presents with progressive
myelopathy over the past 2 months along with incontinence and an inability to ambulate over the past
48 hours. Imaging is as shown. What Is the most accurate statement regarding surgical decision
making in this patient?
A. Regardless of surgery, performance status, serum calcium and albumin levels are the most
important prognostic factors.
B. The presence of a spinal metastasis carries an extremely poor prognosis, therefore no surgery
should be pursued.
C. Aggressive surgical decompression and reconstruion should be performed, as life expectancy in
this patient is > 2 years.
D. Surgical decompression should be pursued in the face of neurologic deterioration during
radiotherapy given its clear benefit relative to risk

Metastatic NSCLC presenting with a spinal cord compression carries a relatively poor prognosis with
a median survival of 8.8 months. As a result it is important to further stratify patients to determine
which ones are best served with surgical decompression/stabilization. Multivariate analysis of
NSCLC patients has demonstrated that performance status, calcium levels, and albumin are the most
significant prognostic factors for survival. Tomita et al. developed a grading system that looked at
tumor histology, state of visceral disease and the presence of other bone metastasis to suggest a
strategy for overall management. Depending on the patients total tumor load and other prognostic
factors an argument can be made for treatment ranging from aggressive surgical resection

19. A 54 year-old female presents 18 months after undergoing instrumented ACDF at C5-6 via a left-
sided approach. She has neck pain and radiculopathy with pseudoarthrosis and instrumentation
failure. She denies hoarseness or swallowing difficulties with the initial surgery. Being right-handed,
you prefer a right-sided approach for her recommended revision surgery. What is the MOST
appropriate management of the surgical approach in this case:
A. Intra-operatve EMG monitoring of the laryngeal muscles
B. Modified barium swallow prior to surgery
C. Right-sided approach without further work-up
D. Laryngoscopic screening prior to surgery
This patient’s preoperative work-up should include laryngoscopy to evalute vocal cord function.
Recurrent laryngeal nerve (RLN) palsy with hoarseness is a well-known complication of ACDF.
However, RLN palsies may be asymptomatic and bilateral palsies can be a catastrophic, therefore it
is inappropriate to proceed with a right-sided approach without first evaluating the patient for an
asymptomatic left RLN palsy. The right- sided approach is acceptable if laryngoscopic exam confirms
lack of clinically silent RLN injury on the left. A barium swallow is inadequate screening because it
does not address RLN function EMG monitoring does not necessarily prevent RLN injury
--

20. A 54 yo male presents with 10 years of progressive back and bilateral radicular L5 leg pain. The
pain is worse when upright and refractory to conservative management. Examination reveals full
strength in his distal lower extremities. Lumbar spine radiographs and MRI are shown in the figures.
Flexion/extension radiographs show 4 mm of motion of L4 on L5. According to the ‘Guidelines for
Management of Degenerative Lumbar Disease’, what is the BEST surgical management strategy:
A. Lumbar decompression
B. Lumbar decompression and fusion
C. Spinal cord stimulator
D. Lumbar fusion
E. Morphine pain pump
According to the Guidelines, lumbar decompession and fusion is recommended for patients with
lumbar stenosis and associated spondylolisthesis. Herkowitz and Kurz published a randomized
controlled trial with improved outcomes for those patients randomized to fusion vs. decompression
alone for lumbar stenosis with degenerative spondylolisthesis. Another randomized trial noted that
the addition of segmental hardware improved fusion rates but did not appear to result in improved
clinical outcomes (Fischgrund et al.,1997). The SPORT trial (Weinstein et al., 2007) also provides
randomized evidence in favor of surgical management degenerative spondylolisthesis. The majority of
patients treated surgically in this trial had associated stenosis at the sponlylolitic level and underwent
fusion with instrumentation.

21. A 55 year-old female presents with back pain, progressive right leg pain, atrophy and weakness.
Exam reveals bilateral non-dermatomal lower extremity hypalgesia and a hemangioma with
hirsuitism over the lumbarspine. Imaging reveals a 10 degree thoracic scoliosis, right sided disc bulge
at L1/2, conus terminating at L4, and a fatty filum terminale (3mm). The treatment of choice is:
A. Conservative therapy (PT, NSAIDS, etc)
B. Spinal deformity correction
C. Realease of filum terminale
D. L1/L2 microdiscectomy
E. Monitoring with serial imaging
Release of fllum terminale and surgical untethering is beneficial in relieving pain and arresting the
rate of neurological decline in cases of symptomatic adult tethered cord syndrome. The purpose of
surgery for occult spinal dysraphism is to release the neural elements from any points of fixation. In
adult surgical series, stabilization or improvement in neurological function is reported to be achieved
in roughly 80-90% of patients. Conservative management with physical therapy, medications, and
simple observation are not appropriate in the setting of progressive neurological decline, as
neurological deficits such as bowel and bladder dysfunction may become irreversible.
Microdiscectomy inadequately addresses the patient’s continued progressive neurological decline.
Symptoms attributed to scoliosis in the presence of a tethered spinal cord will often improve or
stabilize once the tethering lesions are released
--
22. A 58 year old woman presents with neurogenic claudication, MRI demonstrates moderate to
severe stenosis associated with spondylolisthesis at L4-5. She presents for a surgical opinion.
Regarding surgery. which of the foIlowing is true?
A. It prevents potentially catastrophic neurological deterioration
B. It leads to significantly worse long term outcomes due to adjacent segment disease
C. It affords significant short and long term benefits compared to non-surgical treatment
D. It does not influence the clinical outcome in patients with more severe symptoms
E. It must include the use of pedicle screw fusion to improve patient outcomes
There is no evidence to suggest that patients with lumbar stenosis and spondyldisthesis are at
increased risk for catastrophic deterioration. In fact, such deterioration has not been observed in
multiple prospective trials comparing operative and non-operative intervention. Patients with
degenerative spondylolisthesis and stenosis and severe symptoms enjoy substantial improvement
when treated with surgery compared to those treated without surgery. However, patients with mild
symptoms may remain stable or even improve over time. Adjacent segment disease may or may not be
a relevant concern in this patient population but should not be considered a contraindication to
fusion.
--

23. A 58 year-old female presents with a six-month history of progressive mid-back pain. She reports
several falls secondary to her nght lower extremity ‘giving out’. Physical examination reveals normal
strength except for 4+/5 in the right lower extremity. There is sustained, three-beat donus on the right
MRI with and without contrast reveals a partially enhancing intradural, intramedullary mass at T8.
The MOST likely diagnosis is:
A. Glioblastoma
B. Astrocytoma
C. Schwannoma
D. Ependymoma
E. Meningloma
The answer is ependymoma. Thoracic ependymoma is the most common intradural, intramedullary
mass in adults. Schwannomas and menigiomas are usually extra-axial lesions. Spinal cord
astrocytomas are less common and generally enhance homogeneously. Spinal cord glioblastoma is a
rare lesion.
--
24. A 60 year old man presents with 6 months of worsening back pain. He denies any weakness or
numbness. The patient describes the pain as deep and aching. It is located only in the back and does
not radiate. It is worse with activity and improved with rest. He is not tender to palpation on physical
exam. This type of pain may be classified as:
A. Mechanical
B. Malingering
C. Myofascial
D. Oncologic
Pain that is worsened by activity or loading and that improves with rest or unloading defines
mechanioal back pain. Myofascial pain is typically constant, superficial in location, and associated
with palpable tenderness (i.e. trigger points). Pain associated with cancer is often worse at rest. may
be worse at night, and is typically associated with systemic signs. Malingering patients may present
with inconsistent pain, pain out of proportion with the exam, positive Waddell signs, and fits no
traditional pattern.
--

25. A 60 year old man presents with progressive pain at the level of his sacrum over the course of 6
months. MR imaging reveals an 8 cm lesion within his sacrum most consistent with a chordoma.
What is the most appropriate and best treatment option for this patient that is associated with the best
long-term outcome?
A. Intralesional tumor decompression to alleviate the pain and allow for a tissue diagnosis followed
by referral for proton beam radiotherapy.
B. CT guided biopsy of the lesion to confirm the diagnosis followed by en bloc resection of the
lesion including resection of the biopsy tract
C. CT guided biopsy of the lesion to confirm the diagnosis followed by referral for proton beam
radiotherapy
D. Proton beam radiotherapy alone without biopsy to avoid seeding tumor in the biopsy tract given
that the MR appearance of chordomas is highly specific
All sacral tumors must have proper histological confirmation prior to en bloc resection or
radiotherapy. Incisional biopsy or intralesional resection increases the risk of local recurrence of a
chordoma. Therefore, transcutaneous CT-guided trocar biopsy is preferred to open biopsy to
minimize the risk of contamination of normal tissues by tumor. The biopsy tract should be planned to
be included within the subsequent resection margins. En bloc resection of primary spine tumors with
disease-free margins is activable and provides the best long-term survival for patients. Less than en
bloc resection is associated with an almost 100% tumor recurrent rate, even after radiotherapy.
--
26. A 61 year old previously healthy man presents with progressive hand weakness, tingling in his
fingers, and gait instability. Examination reveals atrophy and weakness of the hands, impaired rapid
alternating movements, diminished pinprick sensation in the fingers, diffuse hyper-reflexia, and lower
extremity spasticity. What is the MOST likely diagnosis:
A. Amytrophic Lateral Sclerosis
B. Multiple Sclerosis
C. Cervical Spondylotic Myelopathy
D. Normal Pressure Hydrocephalus
E. Subacute Combined System Disease
The most likely diagnosis is cervical spondylotic myelopathy (CSM). The signs and symptoms
described are consistent with myelopathy. CSM is the most common cause of myelopathy in
individuals over the age of 55 Both static factors, such as acquired or developmental cervical
stenosis, and dynamic factors, producing repetitive injury to the cervical cord, result in direct injury
and initiate a secondary cascade of events induding ischemia, excitotoxicity, and apoptosis. Clinical
manifestations are produced by degeneration of the central grey matter, posterior columns, lateral
columns (especially the corticospinal tracts), and anterior horns cells. Amyotrophic lateral sclerosis
(ALS) leads to progressive degeneration of both upper and lower motor
--

27. A 62 year old with known metastatic renal cell cancer presents with unbearable bad pain and
lower extremity myelopathy after a fall. Her imaging is displayed in the accompanying figures, The
best management strategy of this lesion would be:
A. Kyphoplasty/Vertebroplasty followed by radiotherapy
B. Stereotactic Radiosurgery
C. Conventional external beam radiotherapy
D. Laminectomy and resection of epidural disease followed by radiotherapy
E. Vertebrecomy with posterior stabilization followed by radiotherapy
The most appropriate management Includes vertebrectomy, posterior pedicle screw instrumentation,
and postoperative radiotherapy. Patchell et. al demonstrated in a prospective randomized trial
superior results with circumferential surgical decompression followed by radiation over conventional
radiotherapy in the management of symptomatic metastatic epidural spinal cord compression (Class
1 evidence). Patients who underwent surgical intervention were not only more likely to ambulate
postoperatively, but also more likely to maintain urinary continence and to achieve better pain
control. Stereotactic radiosurgery is not a good option in this case because of the presence of
myelopathy and the high degree of epidural compression which would limit adequate dosing to the
epidural tumor. The pathologic fracture and kyphosis will not respond to radiation alone. Vertebral
augmentation may offer

28. A 65-year old man complains of bilateral lower extremity pain, numbness, and weakness after
standing or walking for 5 minutes. A brief period of sitting or lying down results in complete relief of
symptoms, after which he can walk another 5 minutes. Magnetic resonance image is shown in Figure
1. The clinical syndrome is BEST described as:
A. Neurogenic claudication
B. Diabetic peripheral neuropathy
C. Radiculopathy
D. Sciatica
E. Plexopathy
Neurogenic claudication is the symptom complex most commonly associated with lumbar spinal
stenosis (illustrated in Figure 1). Patients with neurogenic daudication may be asymptomatic at rest,
without neurological deficit. Symptoms typically begin when the patient stands or walks. Back fiexion
typically decreases or delays the onset of symptoms. whereas back extension may exacerbate the
symptoms. For this reason, patients may notice that it is easier to climb stairs than it is to descend.
Neurogenic claudication must be differentiated from vascular claudication, caused by lower extremity
arterial insufficiency. Patients with vascular claudication may present with diminished lower
extremity pulses, non-healing sores on the lower legs or feet, and loss of hair on the calves. Pain
radiating to the groin should always prompt a suspicion of degenerative hip arthritis.

29. A 65-year-old man presents with progressive neck pain, bilateral numb, clumsy hands, spastic gait
and a present Babinski sign. Tongue fasciculations are absent, MR imaging is shown in the figure.
The
MOST likely diagnosis is:
A. Combined systems degeneration
B. Parkinson’s disease
C. Amyotrophic lateral sclerosis
D. Cervical spondylotic myelopathy
E. Cerebral palsy
Cervical spondylotic myelopathy is characterized by a combination of neck pain, numb and clumsy
hands, gait disturbance, sphincter dysfunction, and impotence. Associated physical findings may
include increased lower extremity muscle stretch reflexes, positive Babinski sign, and Lhermitte
phenomenon, all of which are nonspecific signs of myelopathy.
The combination of decreased lower extremity reflexes and positive Babinski sign, although not
absolute, should prompt a consideration of vitamin B12 deficiency (combined systems disease).
Tongue fasciculations and atrophy suggest a process with bulbar involvement (for example, motor-
neurone disease). Cogwheel rigidity is commonly a feature of Parkinson’s disease and combined
systems degeneration. The patients MR imaging is demonstrated in the figure.

30. A 65-year-old man presents 6 months after a motor vehide collision complaining of severe
disabling neck pain. Cervical CT reveals a non-united type II odontold fraclure. The MOST
appropriate management is:
A. Posterior C1-C2 arthrodesis
B. B. Halo immobilization
C. Hard cervical collar
D. Anterior odontoid screw fixation
Posterior C1-C2 fixation/arthrodesis immobilizes the entire C1-C2 complex, and is the treatment of
choice in this condition. Generally, a fracture older than 3 months Is considered a chronic fracture.
Acute type II odontoid fractures do not respond well to external orthrosis (hard collar or halo), and
healing rates are even poorer for chronic fractures. Published clinical series have reported <50%
healing rates for anterior odontoid screw fixation in chronic fractures. Low healing rates are
generally ascribed to pannus formation at the fracture site.

31. A 65-year-old man presents 6 months after a motor vehide collision complaining of severe
disabling neck pain. Cervical CT reveals a non-united type II odontold fraclure. The MOST
appropriate management is:
A. Posterior C1-C2 arthrodesis
B. B. Halo immobilization
C. Hard cervical collar
D. Anterior odontoid screw fixation
Posterior C1-C2 fixation/arthrodesis immobilizes the entire C1-C2 complex, and is the treatment of
choice in this condition. Generally, a fracture older than 3 months Is considered a chronic fracture.
Acute type II odontoid fractures do not respond well to external orthrosis (hard collar or halo), and
healing rates are even poorer for chronic fractures. Published clinical series have reported <50%
healing rates for anterior odontoid screw fixation in chronic fractures. Low healing rates are
generally ascribed to pannus formation at the fracture site.
--
32. A 65-year-old woman presents with severe, progressive back pain is found to have degenerative,
thoracolumbar scoliosis as depicted in the pigures. Which radiographic parameter has the greatest
impact on physical function and disability?
A. Abnormal coronal balance
B. Presence of apical rotation
C. Scoliosis curbe magnitude
D. Positive sagittal balance
E. Presence of single curve

of the choices provided, the radiographic parameter that has been shown to have the greatest impact
on health status measures is positive sagittal balance (measured from C7 to the posterior margin of
the sacrum). Glassman and colleagues assessed the impact of multiple radiographic parameters on
health status measures based on a large population of adult deformity patients. Patients with a
positive sagittal balance had significantly greater pain, decreased physical function, worse self
image, and poorer social function. This underscores the important of achieving sagittal balance when
surgically addressing spinal deformity. This also underscores the importance of ensuring that an
adequate degree of lordosis is provide when performing instrumented lumbar fusions in order to
prevent development of “flat back”
--
33. A 66-years-old white male present after a witnessed fall down six steps complaining of pain and
weakness, worse in arms than legs. Plain radiographs, CT and MRI of the cervical spine are shown in
the pigures. What underlying medical condition does this patient have:
A. Severe Cervical Spondylosis
B. Ankylosing Spondylitis
C. Osteoporosis
D. Diffuse Idiopathic Skeletal Hyperostosis

the answer is Diffuse Idiopathic Skeletal Hyperostosis (DISH). DISH is a common condition
characterized by abundant ossification throughout the body. It occurs most often in the spine with the
anterior longitudinal ligament being most often affected. It affect males predominantly and may occur
in 20% of the population. For the diagnosis, there should be flowing calcification or ossification of at
least four condiguous vertebral bodies in addition to preservation of disc height in the involved areas
and absence of sclerosis of fusion of the SI joints. Other differentiating factors from ankylosing
spondylitis (AS) include syndesmophytes and calcification of the annulus fibrosis and nucleus
pulposus in AS. Among others complications, patiens with DISH are predisposed to spine injures from
minor otherwise trivial mechanisms. Most fractures are of the spine seen in DISH are characterized
as distraction extension injuries. Often there is a transverse shift of the fractured segment as opposed
to the compression fracture, probably due to limited or absent dissipations.
34. A 68 years-old male with a history of prostatic cancer present with low-grade fever and severe
low back pain progressing to include lower extremity numbness. Thoracic CT shows extensive
destruction of the T11 and T12 vertebral bodies with relative sparing of the T11-12 disc space, as well
as a large paraspinous abscess with calcification. Thoracic MRI shows a kyphotic deformity with
enhancing soft tissue and bone extending into the anterior spinal canal and resulting in moderate
stenosis and spinal cord compresion. The MOST likely pathologic process is:
A. Osteoporotic Compression Fracture
B. Discitis – Staph Aureus
C. Pott’s Disease – Tubercolosis
D. Pathologic Fracture – Metastatic Tumor
E. Discitis – Staph Apidermidis
the answer is Pott’s Disease, resulting from tuberculosis. The radiographic description is classic for
Pott’s Disease. Staphylococcal infection is unlikely due to the fact that discitis centered on the disc
space and generally spares the vertebral bodies (opposite to the imaging appearance described in this
case). Pathological fracture due to metastasis is would not explain the presence of a paraspinous
abscess. Osteoporotic compression fracture would not presence with extensive, enhancing epidural
soft tissue extension.
--
35. A 68 years-old woman presents with progressive myelopathy with MRI shown. An isolated
posterior approach is contraindicated in this patient in the presence of:
A. Fixed kyphotic deformity
B. Posterior ligamentous hypertrophy
C. Subluxation at C3/4 and C4/5
D. Ventral compressive pathology
E. Compressive pathology across multiple levels

an isolated posterior approach is contraindicated due the fixed kyphotic deformity. Among the listed
factors, appropriate assessment of the cervical alignment is most important when determining if a
posterior approach is appropriate. The posterior approach is ideally suited for patiens demonstrating
a degree of lordosis. The presence of a straight or kyphotic alignment will limit dorsal migration of
the spinal cord and potentially cause the spinal cord and nerve roots to drape across ventral
pathology, leading to further neurologic compromise
--
36. A 70 years-old male with a type II dens fracture has non-union of the dens despite 6 month of halo
fixation. His new cervical CT scan demonstrates non-union of the dens with 6 mm of posterior
displacement. Cervical x-rays reveal that anatomic reduction of the fracture is not possibble. No other
associated fractures are identified. He is otherwise neurologically intact and is in good medical
condition. What is the most appropriate treatment option at this time?
A. Continued halo management for an additional 3 months
B. Management in a hard cervical collar for 3 month
C. Removal of halo fixation and observation
D. Anterior odontoid screw fixation
E. Posterior C1-2 instrumented arthrodesis
the most appropriate management would be to perform posterior C1-2 instrumented arthrosis. The
reported rate of chronic nonunion fractures of the dens after halo fixation in the elderly is around
28%. A posterior fixation would provide the best rate of fusion as compared to an anterior odontoid
screw fixation in the setting of chronic nonunion of dens. The reported rate of fusion from a posterior
C1-2 arthrodesis is approximately 85%. Contraindications to anterior odontoid screw fixation
include: disruption of the transverse ligament, concomitant atlantoaxial joint injuries, fracture line
parallel to screw trajectory, cervical kyphosis, barrel chest habitus, obesity that would make the
screw trajectory not possible. Anterior screw fixation should not be used in patient with nonunion
fractures in which fractures healing and/or fixation will be impaired. Furthermore, the 6 mm of
posterior displacement would make the posterior approach superior to an odontoid screw.
--
37. A 72 years old woman has been diagnosed with osteoporosis, with a T-score of -2.6 on a recent
central DXA scan. She has recently been started on supplemental calcium (1500 mg daily) and
bisphosphonate. Which one of the following would be the most appropriate next step in management
of her osteoporosis?
a. Screening urinary calcium levels
b. Doubling the calcium supplement
c. Repeating bone density testing in 2 years
d. Measuring hormonal levels
e. Obtaining x-rays of the lumbar spine
in asymptomatic postmenopausal women with osteoporosis, many physicians monitor changes in bone
mineral density (BMD) by central DXA every year or two during pharmacologic therapy for
osteoporosis. It is important to note that drugs may decrease a patient’s risk of fracture even when
there in no apparent increase in BMD. As with most test, BMD has some precision error. Therefore,
changes of less than 2 to 4% in the vertebra and 3 to 6% at the hip from test to test can be due to the
precision error of the method. Repeat bone density measurements for monitoring disease progression
in response to therapy should generally be performed after 1 or 2 years. Two years is consistent with
guidlines developed by the Centers for Medicare & Medicaid Services (CMS).
38. A C1-2 transarticular screw is most likely to result in injury to the vertebral artery when
misdirected:
A. Medially
B. Caudally
C. Laterally
D. Anteriorly
E. Cranially
the vertebral artery is most vulnerable to injury during C1-2 transarticular screw placement due to
drill misdirection caudally. The vertebral foramen lies caudal to the pars of C2, and anatomic
variations of the foramen can place the vertebral artery at risk during placement of a transarticular
screw. Cranial misdirection of the screw could possibly affect the occiput-C1 joint altough this
occurrence is extremely uncummon. Medial placement of a transarticular screw could result in dural
laceration, with or without spinal cord injury. Placement of a transarticular screw too far anteriorly
has been reported to cause hypoglosal nerve dysfunction in few case reports.
--
39. A 54 years old male presents with a three day history of bladder problems. He states that he does
not feel his bladder filling, and that urine sometimes leaks out. Neurologic exam is normal with the
exception of hypoactive reflexes. His CT myelogram is displayed below. What is the most appropriate
treatment for this patient?
A. Endovascular therapy with particulate embolic agents
B. A ventral approach, carpectomy and recection of the lesion
C. A posterior approach, laminectomy, durotomy, and ligation of draining vein
D. Continued observation, physical therapy and lumbal strenghtening
E. Endovascular embolization followed by radiosurgery

the patient has a Type I spinal malformation, or a spinal arteriovenous fistula. The fistula lies within
the dura and is an abnormal connection between the radicular vein and artery lying dorsally on the
spinal cord making a posterior approach preferable. His problem is caused by venous engorgement in
the spinal cord from backflow of venous blood through the radicular draining vein into the coronal
venous system on the dorsal aspect of the spinal cord. An arterial steal phenomenon may also be
contributing to the problem, altough this is likely less significant
40. A normal intraoperative electromyographic study during lumbar fusion surgery has been shown to
the BEST corelate with:
A. Improved fusion rates
B. Improved patient outcomes
C. Lack of a neurological injury
D. Pedicle fractures
intraoperative electrophysiological monitoring during pedicle screw placement is useful as a
diagnostic test, as a normal evoked EMG response is highly predictive of intrapedicular screw
placement in the pedicle adjacent to the monitored root. The use of such monitoring has, however,
never been shown to devrease the incidence of pedicle breach during screw placement, decrease the
incidence of neurological injury, or improve patient outcomes. An abnormal EMG response,
moreover, is not highly predictive of a neurological injury (because of a high false-positive rate of
EMG abnormalities of this setting)
--
41. A patient awakens from left-sided costotransversectomy at T10 for resection of a ventral
metastatic tumor with complete paraplegia and loss of pain and temperature sensation. His sensation
to light touch in the lower extremities is spared. Post operative MRI reveals no evidence of spinal
cord compression or hematoma. What the most likely diagnosis?
A. Posterior cord syndrome
B. Anterior cord syndrome
C. Central cord syndrome
D. Brown-Sequard Syndrome
the most likely diagnosis is anterior cord syndrome. The sacrifice of the left T10 nerve root likely
compromised the artery of Adamkiewicz. This artery supplies the anterior 2/the most likely diagnosis
is anterior cord syndrome. The sacrifice of the left T10 nerve root likely compromised the artery of
Adamkiewicz. This artery supplies the anterior 2/3 of the spinal cord. It commonly arises at T10 on
the left but may arise anywhere from T7-L4. It may be seen on the right in 17% of patients. When
planning o posterolateral approach for ventral access to the spinal cord, a spinal angiogram may be
helpful in identifying the artery of Adamkiewicz to prevent a vascular insult to the cord when
sacrifying a nerve root.
--
42. A patient present with a classic Type II odontoid fracture. A MRI and Flexion-Extension Dynamic
Radiographs of the cervical spine are consistent with disruption of the transverse ligament. The
MOST appropriate treatment is:
A. Anterior odontoid screw fixation
B. Halo immobilization
C. Occipital-cervical fusion
D. Hard cervical collar
E. Posterior C1-2 fixation
Disruption of the transverse ligament indicates an unstable C1-2 complex, requiring C1-2 fixation.
Because the instability is ligamentous is nature. Treatment must be based on functionaly fusing the
unstable segment. Hard cullar and halo treatment immobilize bony fractures for healing but are
inadequate to address ligamentous instability. Anterior odontoid screw fixation simply
reapproximates the fractured odontoid peg to the C2 body and is inadequate to address an unstable
C1-2 segment. Occipital-cervical fusion sacrifies adjacent motion segments needlessly, including the
occiput-C1 motion segment which is functionally critical.
--
43. A patient present with bilateral nondisplaced fractures through the C2 pars interarticularis
(hangman’s fracture). Flexion-extension dynamic cervical radiographs show < 2mm motion and no
significant deformity. The most appropriate treatment modality is:
A. Anterior C2-3 discectomy and fusion
B. Posterior C1-2 fixation
C. Posterior C1-3 fixation
D. External immobilization
E. Occipito-cervical fixation
the majority of hangman’s fractures are considered stable fractures. The literature consistently
reports exellent healing rates with external orthosis, both hard collar and halo immobilization.
Operative fixation is reserved for unstable fractures (i.e > 5 mm displacement or gross malalignment)
--
44. A spinal cord injured patient’s examination reveals the presence of voluntary but non-functional
strenght in the lower extremities (more than half of the muscles have less than antigravity strenght).
Sensation to pain, temperature, and proprioception is markedly diminished distal to the injury. The
patient’s american spinal injury association (ASIA) impairment scale grade is:
A. E
B. C
C. A
D. D
E. B

the ASIA grading system was developed to standardizet the reporting of spinal cord injuries. It is a
modification of the Frankle scale. Grades range from A to E. A lessions are characterized by no
motor or sensory function detected below the level of the lession, including the sacral segment. Grade
B is characterized by no motor function detected below the level of the lession and some preserved
sensory function below the level of the lession. Grade C patients characteristically have some
voluntary motor function preserved below the level of the lession, but this function is too weak to
serve any useful purpose (define as more than half of key muscle having a strenght grade of 3 or
less). Sensation may or may not be preserved. Grade D patients have functionally useful voluntary
motor function below the level of the injury. While grade E patients have normal strength and
sensation, altough abnormal reflexes may persist based upon the above grading scheme, this patiens
injury would be catagorized as grade C
45. A thirty six years old female presents to the emergency room with a two day history of
difficulty with urination, anterior tibialis weakness and diminished rectal tone. Her MRI is attached.
What is the most important surgical goal?
A. Instrumented stabilization and fusion
B. Generous posterior decompression
C. Lysis of adhesions in the cauda equina
D. Resection of the mass
E. Sectioning of the filum terminale
the patient has a terminal lipoma of the conus and filum terminale that is tethering the spinal cord.
The lession is not malignant, nor is it likely to grow significantly. The patient’s symptoms are most
likely related to spinal cord tethering. Her best chance of improvement and prevention of further
neurological deterioration is to completely untether the cord, including section the filum terminale
and detachment of the lipoma from adjacent dura. Some of the lipoma may need to be removed to
accomplish an untethering, but it does not require resection. Attempts at comprete resection puts the
patient at risk for neurologic deficit as it is likely adherent to the neural elements.
--
46. A twenty four years old woman present with right arm and leg weakness and left cranial nerve VI
palsy following a high-speed motor vehicle accident. Her lateral cervical spine x-ray shows upper
cervical prevertebral soft tissue swelling and a basion-dental interval of 16 mm. Her head CT is
negative for intracranial hemorrhage. What is the recommended treatment of this injury?
A. In situ arthrodesis and immobilization in a 4-poster brace
B. Traction followed by immobilization in a halo
C. Immobilization in a hard collar
D. Internal fixation and arthrodesis
E. Emmobilization in a halo
the most appropriate treatment option is internal fixation and arthrodesis. Traumatic atlanto-
occipital dislocation frequently results in death at the time of injury. Surviving patients may present
with a normal neurological exam, but often present with hemiparesis or quadriparesis. Cranial nerve
palsies may be seen. This injury is extremely unstable and patients frequently deteroriate without
timely occipitocervical instrumentation.
--
47. According to the BEST medical evidence available regarding lumbar fusion, the addition of
pedicle screw fixation to a single or double level posterolateral fusion performed for chronic low back
pain due to degenerative disease without deformity or neurological deficit is associated with which of
the following outcomes:
A. Unchanged complication rate
B. Higher fusion rate
C. Lower overall cost
D. Shorter hospital stay
E. Improved patient outcomes
according to the best medical evidence available, the placement of pedicle screws as an adjunct to
posterolateral fusion performed for low back pain in patiens without deformity or radiographic
instability improves fusion rates, but does not improve patient outcomes. There is compelling
evidence that the addition of pedicle screws increase cost and complications.
--
48. According to the guidlines for the Performance of Fusion Procedures for Degenerative Disease of
the Lumbar Spine, the literature supports the use of fusion in lumbar decompression surgery for
degenerative stenosis when associated with:
A. Conjoined nerve root
B. Focal disc herniation
C. Spondylolisthesis
D. Spina bifida
E. Congenital stenosis
medical evidence does not support the addition of fusion to decompressive surgery in patient with
degenerative lumbar stenosis unless there are additional structural problems such as
spondylolithesis, for which there is Class I evidece. Some Class III evidence supports the addition of
fusion in patients with no deformity but with instability demonstrated on flexion and extention x-rays
and on patiens undergoing wide laminectomies (with significant facet disruption). There is no
substansial evidence available on the addition of fusion to decompression for patients with stenosis
and congenital stenosis, spina bifida, conjoined root, or focal disc herniation.
--
49. According to the guidlines for surgical management of degenerative spinal disease, the short and
long term economic impact of lumbar fusion surgery for degeneratve spinal disease is:
A. Positive in the short term and negative in the long term
B. Negligible in the short term and positive in the long term
C. Negative in the short term and negative in the long term
D. Negative in the short term but positive in the long term
E. Positive in the short term and positive in the long term
the economic impact of lumbar fusion surgery is negative in the short-term, partiqularly if
instrumentation is involved. In the long term, however, there are beneficial economic effects,
partiqularly with respect to return to employment data. The impact is not negligible. It is quite costly
up front but often has a significant long term beneficial impact. In a carefully selected patient pool the
initial negative impact of fusion surgery is outwighed by the beneficial effects over time.
--
50. After a type II odontoid fracture, the function of which ligament/membrane most strongly
influences treatment options:
A. Alar
B. Tectorial
C. Apical
D. Transverse
E. Anterior longitudinal
rupture of the transverse ligament (also known as the cruciate ligament) allows translation of C1 on
C2, evidenced by an increase in the anterior dental interval. Incompetence of this ligament is a
contraindication to odontoid screw fixation and is associated with delayed instability. Incompetence
may be diagnosed based on either increase in the anterior dental interval (>3mm in adults is
considered abnormal) or MRI findings. The other ligaments (alar, apical, interspinous and anterior
longitudinal) are important in determining the degree of stability, need for bracing or internal
fixation, prognosis for healing and other factors, but individually are less determinative of the spesific
operative approach, in particular the need for C1-2 bony fusion.
--
51. An eleven years old boy is brought to clinic by his parents with a one week history of neck pain.
On questioning, the boy denies any falls or other accidents. He has no significant past medical history.
A review of system is non-contributory. On physical exam, he is well-develoved, well-nourishd
young male with a normal posture. He is slightly tender to midline palpation of the servical spine.
Cervical range of motion is somewhat limited. His neurological exam is normal. His vital signs are
normal, and he is afebrile. An MRI of the servical spine was obtaine by the boy’s pediatrician. The
lateral view is displayed. What is the most likely diagnosis?
A. Paget’s disease
B. Metastatic cancer
C. Hurler’s syndrome
D. Pott’s disease
E. Eosinophilic granuloma

Eosinophilic Granuloma is the spine is a benign, non-infectious disease process which typically
present as a solitary lession, restricted to the vertebral body without involvement of the disc space
and/or posterior elements. A systemic workup of such patients is generally advisable.
52. Based on published medical evidence review, discography may BEST be used as a diagnostic test
for which clinical purpose?
A. Functional Imaging relevant to patient selection for lumbar fusion
B. Repair of annular tears
C. Visualization of clinically relevant disc space abnormalities
D. Prediction of outcome following lumbar interbody fusion
E. Diagnosis of discogenic low back pain
discography does not have adequate sensitivity or spesificity as an independent predictor of outcome
following lumbar fusion surgery. Discography does not allow visualization of clinically relevant disc
abnormalities not visualized by non-invasive axial imaging techniques. When employed using
concordant pain response as an outcome measure, however discography may serve as a functional
study of the lumbar spine relevant to the identification of pain arising from individual degenerative
discs.
--
53. Class I medical evidence supports the use of lumbar fusion in patients with degenerative low back
pain, without stenosis or spondylolisthesis, under which of the following circumstances:
1. Acute severe axial back pain
2. Protracted pain responding to medical management
3. Protracted pain arising from 3 or more levels
4. Acute pain refractory to epidural injections
5. Protracted pain refractory to multi-modality medical management
Class I medical evidence supports the use of fusion surgery in the treatment of patients with
degenerative spine disease who have been carefully selected and who have had pain that has been
refractory to the best medical management. Such patients do better with surgery than do patients
treated with conservative therapy. Stenosis or spondylolisthesis need not be a component of the
disorder. Precise definition of selection criteria remains elusive. Degenerative disc disease is
ubiquitous. Multi-level changes make designation of the pain-generator problematic. Patients may
respond to conservative measures despite profound multi-level degenerative changes on radiographic
studies.
--
54. During a retroperitoneal approach to the lumbar spine, what structure runs along the medial aspect
ot the psoas muscle ang lateral aspect of the spine?
1. Ilioinguinal nerve
2. Ureter
3. Sympathetic trunk
4. Genitofemoral nerve
5. Aorta
the sympathetic trunk runs medially along the medial border of the psoas. The ilioinguinal nerve
emerges along the upper lateral border of the psoas to the quadratus lumborum. The genitofemoral
nerve travels more laterally along the psoas. The ureter is usually adherent to the posterior
peritoneum and generally falls away from the psoas and spine during the dissection and exposure, as
does the aorta.
--
55. Following anterior cervical discectomy and fusion (ACDF), which of the following complications
is the most common?
1. Dyshagia
2. Homer’s syndrome
3. Thoracic duct injury
4. New radiculopathy
5. Hoarsness
Temporary or persistent dysphagia can occur in up to 80% of post-operative ACDF patients.
Improvement may take many months. Dysphagia appears to be secondary to local response to
traction and manipulation rather than locking plate profile (this was not an uncommon complication
of anterior cervical surgery prior to the employment of locking plate system). An overly proud
construct conformity certainly add to the syndrome. Other complication and syndrome are thankfully
more rare altough overall morbidity for the procedure can approach 25%. Esophageal injury occurs
in less than 1% of cases as does Homer’s syndrome, vertebral artery injury, and thoracic duct injury.
New neurologic deficits can occur in up to 2% of cases. Recurrent laryngeal nerve injuries (resulting
in hoarseness) occur in 1-2% of cases.
--
56. Harrington distraction rods were commonly used for the correction and stabilization of scoliosis.
At which of the following locations is a harington rod most likely to fracture:
1. The upper hook/rod (point A)
2. The middle of the shaft (point B)
3. The lower hook/rod (point E)
4. The middle ratchet (point D)
5. The proximal ratchet (point C)

the proximal ratchet (point C) is the point of maximum stress application. Stress application is
defined by the bending moment and strength of the rod, which is proportional to the cube of the
diameter of the rod. The proximal ratchet (point C) is a transition point from the diameter of the
shaft to the smaller diameter of the ratchet. Because the strength is define by the cube diameter, even
a small changes in the diameter will significantly affect the strength of the rod. The bending moment
at either end of the rod are too short to define a large enough stress to fracture the rod. The rod
would be unlikely to fracture at mid-shaft due to the fact that it’s diameter is much greater than the
proximal ratchet.
--
57. In the spine, which tumor is most commonly located in an intradural/extremedullary location:
1. Astrocytoma
2. Hemangioblastoma
3. Ependymoma
4. Adenocarcinoma
5. Schwannoma
the most common intradural/extremedullary neoplasm are the schwannomas and neurofibroma.
Schwannomas are composed of neoplastic Schwann cells, whereas neurofibromas are composed of a
mixture of neoplastic Schwann cells, fibroblasts, and perineural cells. Other less common
intradural/extremedullary spinnal lesions include meningiomas, and ependymomas are most
commonly intramedullary lesions. Carcinomas are most commonly extradural spinal column lesions.
58. The first priority in the overall assessment of a trauma patient with an acute cervical spine fracture
is:
1. Ongoing hemorrhage
2. Neurological deficit
3. Spinal instability
4. Blood pressure
5. Airway integrityd
altough spinal instability, neurological deficit, and control of hemorrhage certainly are important
issues in the management of an acute cervical spine fracture, ATLS protocol dictates that assessment
of airway patency takes precedence. This is followed by assessment of circulation/control
of hemorrhage, and performance of a brief neurological examination. During initial management of
the trauma, it should always be presumed that the patient harbors an unstable spinal injury.
Assessment of spinal stability and acquisition of additional studies should occur after the secondary
survey has been performed.
--
59. The most likely cause of decreased fusion rates in smokers undergoing lumbar spinal arthrodesis
is:
1. Decreased overall nutritional health
2. Cardiopulmonary complication from COPD
3. Peripheral vascular disease
4. Inhibition of bone formation by nicotine
5. Greater noncompliance with postoperative treatment recomendations
a number of clinical studies have reported lower fusion rates in smokers. Animal studies have
demonstrated the effect of nicotine on lower rates of fusion. While the other choices listed may
contribute to the failure of successful arthrodesis in patients, nicotine appears to be the primary
factor involved
--
60. The right vertebral artery is injured during posterior C1-2 transarticular screw fixation, while
placing the first screw. The MOST appropriate surgical management strategy is:
A. Place the left screw
B. Remove the right screw
C. Extend the fusion to the occiput
D. Use only right sided instrumentation
Unilateral vertebral artery injury may be tolerated, whereas bilateral injury is generally fatal. If
bleeding can be controlled at the time of surgery, the screw on the side of vertebral artery injury can
be placed or left in place. Postoperative arteriography is recomended to evaluate for propagating
thrombus or dissection. If interventional techniques are available and thrombus or dissection is
identified, temporary and then permanent baloon oclusion may prevent embolic complications. Once
a single vertebral artery is compromised, no attempt should be made to place to contralateral screw
and potentially endanger the contralateral vertebral artery. There are reports in the literature of
fatality occurring as a result of brainstem infarction after bilateral vertebral artery compromise
during C1-2 transarticular screw placement.
--
61. The use of recombinant bone morphogenetic protein in the performace of a lumbar fusion is
currently supported by randomized controlled clinical trials for which fusion approach/type:
A. Posterior lumbar interbody fusion procedures
B. Anterior lumbar interbody fusion procedures
C. Non-instrumented posterolateral lumbar fusion procedures
D. Instrumented posterolateral lumbar fusion procedures
a randomized, controlled clinical trial comparing recombinant human BMP2 to autograft has been
performed in the context of anterior lumbar interbody fusion procedures employing a threaded
titanium cage. The use of rhBMP2 was associated with fusion rates and clinical outcomes at least as
good as those obtained with autograft. There have been no published randomized trials and very few
cohort studies describing the use of BMP’s with other fusion techniques. All such uses are considered
off-label uses.
--
62. What are the current evidence-based guidelines regarding posterolateral lumbar fusion for patient
with radiculopathy due to intervertebral disc herniation?
A. Recomended in patient with recurrent disc herniation without evidence of instability, deformity, or
chronic low back pain.
B. Not recomended because interbody fusions are associated with better outcomes.
C. Recomended for patients with large para-central disc herniation.
D. Do not recomended because simple decompression is associated with better outcomes.
E. Recomended for patient who fail to improve with six to eight weeks of conservative management.

patients with radiqulopathy due to disc herniation who do not improve with measures are good
candidates for decompressive surgery. However, the outcome of lumbar disc surgery for
radiculopathy is not improved by the addition of a posterolateral fusion in the vast majority of
patients. Therefore, the routine use of posterolateral fusion is not recommended as a primary
treatment for patients with radiculopathy.

--
63. What grade of spondylolithesis is depicted by the x-rays:
A. V
B. IV
C. III
D. II
E. I

the lesion depicted in figures 1 and 2 is most consistent with a grade II isthmic spondylolisthesis.
Isthmic spondylolisthesis occurs as a result of bilateral fractures of the pars interarticularis.
Radiography shows 25% to 50% anterior listhesis of L5 on S1. That classifies this lesion as a
Meyerding grade II. The meyerding grading system (reference meyerding HW. Spondylolisthesis.
Surg Gynecol Obstet 54:371-377, 1932) is widely used and denotes the grade of slippage in 25%
increments. Therefore, a 0 to 25% slip is a grade I, a 25% to 50% slip is a grade II, etc. Isthmic
spondylolisthesis rarely produces central spinal stenosis because the bony roof of the spinal canal
remain dorsally situated because of either elongation or frank fracture of the pars interarticularis,
essentially disconnecting it from the vertebral body. The nerve root exiting at the level of the
malalignment is most prequently affected. In this case, the L5 nerve root, exiting below the L5 pedicle
and the body of sacrum. Therefore, L5 radiculophaty and/or low back pain would be common
findings.
64. What is the most common adverse event following laminoplasty for cervical spondylotic
myelophaty?
A. Loss of cervical range of motion
B. C5 root palsy
C. Inadequate cord decompression
D. Progressive cervical kyphosis
E. Accelerated adjacent level deterioration
laminoplasty has been advocated over laminectomy as a means of achieving adequate dorsal
decompression in the cervical spine, while decreasing the risk of progressive kyphosis in patients with
lordotic or straight cervical spines. However, the literature reports a range of 30%-50% loss of
cervical range of motion postoperatively in selectes series. While C5 root palsy is a noted
phenomenon occuring in cervical decompression for CSM and OPLL, it’s pathophysiology is poorly
understood and the rate of occurence has been reported in the range of 10-15% in selected series.
--
65. What is the recomended insertion torque for halo pins in adults?
A. 8 inch pounds
B. 6 inch pounds
C. 4 inch pounds
D. 10 inch pounds
E. 12 inch pounds
halo pins inserted with 8 in-lb insertional torque demonstrate significantly less loosening and pin site
infection with cynical loading than pins with 6 in-lb.
--
66. What radiographic finding is a contraindication to cervical laminoplasty?
A. Spinal cord signal change
B. Ossified posterior longitudinal ligament (OPPLL)
C. Multilevel cervical spondylosis
D. Cervical kyphosis
E. Congenital cervical stenosis
A contracindication to performing a cervical laminoplasty is a patient with cervical kyphosis. The
concern for kyphotic patients after laminoplasty is worsening of the kyphosis. Moreover, in kyphosis,
a laminoplasty will not allow the cord to migrate posteriorly due to the bow-stringing effect of the
kyphotic deformity. Cervical laminoplasty is appropriate for patient with multilevel cervical
spondylosis and can be used in patients with spinal cord signal change. The procedure is indicated as
a possible treatment option for patient with OPLL and congenital cervical stenosis. The ideal
candidate will have a good cervical lordosis and be free of axial neck pain. Patients with
straightening of the cervical spine may also be candidates for laminoplasty.

--
67. Which of the following is considered a major risk factor for osteoporosis by the National
Osteoporosis Foundation?
A. Prior falls
B. High body mass index (BMI)
C. Current smoking
D. Use of NSAIDs
E. Alcohol use of 1 drink per day
According to the National Osteoporosis foundation, the major risk factors for osteoporosis (and
related fractures) in Caucasian postmenopause women are:
-personal history of fracture as an adult
-history of fragility fracture in a first-degree relative
-low body weight (<about 127 lbs)
--
68. Which of the following is the most caudal level(s) effectively immobilized by a standard
thoracolumbosacral orthosis (TLSO)?
A. T9-10
B. L1-2
C. T11-12
D. L3-4
E. L5-S1
Without more distal immobilization, such as a tigh extension, the lower two lumbar segmentss
generally show the same or more mobility with a TLSO. Furthermore, studies have demonstrated
more notable reductions in motion at L3-4 than at L4-5 without the use of a thigh extender with a
TLSO.
--
69. Which of the following most accurately defines, from a biomechanical perspective, the location of
rod fracture as depicted in figure 1? The point of:
A. Maximum stress application
B. 3-point bending
C. Narrowest diameter
D. Greates flexibility
E. Maximum strain application

stress is a measurement of force per unit area and in this case defined mathematically as the bending
moment (torque) divided by the strength of the rod. Strength is a measurement of a material’s ability
to resist stress and is proportional to the cube of the diameter of a rod. The point of maximum stress
application, typically located at the middle of a long construct, is the usual fracture point. This rod
has a constant diameter. If it tapered, or had a transition point, this would likely be a point of fracture
for a constant stress application.
1. A 21 years old male presents to the emergency department after being involved in a gunfight. His
GCS is 15 with a non – focal examination but several bullet holes are noted in his scalp with
protruding cerebral tissue. His CT Scan of the head is shown in the figure. What is the BEST
definitive management strategy for this injury :
A. 14 – day course of antibiotics
B. Bedside laceration repair
C. Surgical removal of all bullet fragments
D. Cranioplasty and removal of accessible fragments
E. Surgical durotomy repair

Given the easily accessible, non – eloquent confused brain and large elevated fracture, surgical
removal of the accessible bullet fragments is the procedure of choice. There are reports of their
superficial only (local irrigation and wound closure) or aggressive approaches (craniotomy and
thorough debridement of injured brain and the gun shot tract for removal of deep fragments) to
treatment. However, the best evidence supports an intermediate approach in the majority of cases, in
which readily accessible bone and missile fragments are removed in addition to non – viable brain. In
the above case a there is a large elevated fragment of frontal bone in addition to numerous
entrance/exit wounds seen on the patient’s scalp.
2. A 24 years old patient with gunshot wound to C3 is resuscitated after a PEA code and found to
have fixed and dilated pupils, absent corneal reflexes, and absent cough and gag reflexes. A head CT
reveals diffuse cerebral edema with loss of grey – white differentiation. He is normothermic and
normotensive and his urine toxicology screen and electrolyte panels are within normal limits. Which
component of the brain death examination cannot be used in this patient to evaluate for brain death?
A. Cold caloric testing
B. Nuclear cerebral blood flow study
C. EEG
D. Apnea testing
E. Transcranial Doppler Ultrasonography
An apnea test in apatient with a C3 level spinal cord injury will be confounded by his spinal cord
injury and will not be a reliable way to assess for brain death.
In a patient who is normothermic and normothensive and whose urine toxicology screen and
electrolyte panels are within normal limits. The absence of brain stem reflexes (including cold caloric
testing) are consistent with a clinical diagnosis of brain death. In the absence of confounding factors
(such as that seen in this case), a confirmatory apnea test would be sufficient to diagnose brain death.
However, in light of this confound, other confirmatory tests are required to either (1) confirm the
absence of cerebral blood flow (e.g. transcranial Doppler Ultrasonography or nuclear cerebral blood
flow studies, or angiography)
--
3. A 25 years old male is thorought to the emergency department after a motor vehicle crash. His
blood pressure is low, requiring pressors. His GCS score is 5. A ventriculostomy is inserted and his
intracranial pressure is found to be 35 mmHg. What is the most appropriate immediate treatment for
lowering his ICP?
A. CSF drainage
B. Mannitol
C. Barbiturate Coma
D. Calcium channel Blocker
E. Hyperventilation
The most appropriate treatment for this patient’s intracranial hypertension is drainage of CSF. This
is the only option listed that will not also have the undesirable effect of compromising cerebral
perfusion and thereby potentially aggravating cerebral ischemia.
Barbiturate coma may be effective for lowering ICP in many patients through its suppression of
cerebral metabolism with a consequent decrease in cerebral blood flow and blood volume. However,
its major adverse effect is hypotension. As a general rule, barbiturates should not be used in patients
in whom it is already difficult to maintain a normal blood pressure. Additionally, several other
measures to lower ICP are generally employed prior to the use of this intervention. Calcium channel
blocker are not used clinically to
4. A 30 year old male is brought to the emergency department after a motor vehicle crash. His GCS
score is 13. He begins to have seizure activity in the emergency department and the seizure is
continuing after several minutes. What is the pharmacologic treatment of choice for the seizure?
A. Phenobarbital
B. Pancuronium
C. Phenytoin
D. Lorazepam
E. Paraldehyde
The answer is lorazepam. The recommended initial pharmacologic treatment of persistent seizures
consists of lorazepam (Ativan), 0.1 mg/kg up to 4 mg given over two minutes. Benzodiazepines are
preferred as initial treatment because of their high efficacy and rapidity of action. Lorazepam is
preferred over diazepam (Valium) because of the greater duration of its anti seizure effect (12 – 24
hours vs 15 – 30 mins). Anticonvulsants such as phenytoin or Phenobarbital can be started in
addition to the forst line treatment for prevention of subsequent seizures. Pancuronium is a skeletal
muscle relaxant. Because they have no antiepileptic effects. These type of drugs are not indicated as a
primary treatment for seizures, and in fact they make the clinical detection of ongoing seizures more
difficult. Their only role in these patients is to
--
5. A 40 year old male develops left facial droop and left hemiparesis 48 hours after a motor vehicle
accident despite initially being neurologically intact after the accident. A non – contrast head CT at
the time of detoriation is negative. The most appropriate net diagnostic evaluation is :
A. Cerebral perfusion study
B. CT of the cervical spine
C. MRI of the cervical spine
D. CT angiogram of the cervical spine
E. ICP monitoring
This patient’s clinical course (delayed neurological deficit post – trauma in the setting of a negative
head CT) is most consistent with a vascular dissection in the neck or at the skull base. For carotid
injury, this may become evident within hours of the injury. Vertebrobasilar dissections typically
present 2 – 3 days after the injury. The most sensitive and specific study would be a digital
subtraction angiogram (not an option). Followed by a CT angiogram. This study allows evaluation of
both the carotid and vertebral systems from their origin through the skull base, and is sensitive to
both dissection and external compression from fracture – dislocations or hematoma. Carotid duplex
study is not sensitive for injury to the vertebral system, nor will it demonstrate vascular injury at the
skull base. Other anatomic imaging of the neck(CT/MRI) may demonstrate a traumatic cause for the
vascular injury, but will not specifically demonstrate the vascular abnormality. A cerebral perfusion
study may demonstrate the areas at risk due to embolic events, but will not demonstrate the
underlying cause. ICP measurements is superficious since the patient is awake anda alert.
6. A 52 year old restrained driver presented after a motor vehicle accident with an L2 sensory level,
4/5 strength in his proximal and 4-/5 strength in his distal lower extremities and a severe L2 fracture.
What is this patient’s ASIA impairment Scale score (modified Frankel score) :
A. B
B. D
C. C
D. E
E. A
The patient here has sustained an ASIA D injury, where motor function is preserved below the
neurological level, and more than half of the key muscles below the neurological level have a muscle
strength of at least grade 3. An ASIA A injury is complete, an ASIA B injury is incomplete where
sensory function is spared without motor function except for preservation of the sacral segments. An
ASIA C injury has a motor function preserved below the neurological level, and more than half of the
key muscles below the neurological level have a muscle strength less than grade 3. ASIA E is normal
strength
--
7. Comminution of the occipital condyle is most often associated with which cranial nerve palsy?
A. X
B. IX
C. VII
D. XI
E. XII
The cranial nerve most comon affected by occipital condyle fracture is the CN XII due to the close
relationship of the hypoglossal canal to the occipital condyles. Cranial nerves X, IX, and XI (in order)
are the net most common cranial nerves affected because they course through the jugular foramen.
--

8. The GCS score for the patient shown in the video is :


A. 3
B. 6T
C. 5T
D. 10T
E. 7T
The patient in this example is intubated orally and therefore receives a 1T for verbal response. The
patient does not open his eyes to verbal or painful stimuli (1) but does localize briskly with his right
upper extremity (5). The total GCS score is 7T. The GCS is the most widely used scoring system for
quantifying the level of consciousness after traumatic brain injury (TBI). It is used primarily because
it is simple, has a high degree of interobserver reliability, and correlates with outcome after seere
brain injury. The GCS is shown in Figure 1. The examiner determines the patient’s best eye opening
response, best verbal response, and best motor response. The total score (range 3 – 15) re[resents the
sum of the numeric scores of each of the categories. Use of the GCS has limitations. If apatients has
an ETT in place, verbal scores cannot be assessed. For this reason, many prefer to document the
score by its individual components, so a non – intubated patient with a GCS score of 15 might be
documented as a follows : E4 V5 M6. An intubated
--
9. The neurological examination of a34 year old patient who suffered an MCA stroke 20 hours ago
declines acutely from alert but hemiplegic to GCS 4 (E1, M3) despite maximal medical management.
A head CT reveals a large right MCA infarct with 2 cm of midline shift and cisternal effacement. The
most appropriate management of this patient is :
A. Endovasculae clot retrieval
B. Endovascular TPA administration
C. Heparin bolus and infusion
D. Ventriculostomy
E. Decompressive hemicraniectomy
In a young person with non – dominant MCA infarct less than 24 hours old who is rapidly
deteriorating due to herniation who fails conservative measures, decompressive craniectomy is the
only intervention that can acutely reverse the subfalcine herniation. Recent studies suggest
decompressive craniectomies in the setting of stroke may be beneficial even in patients beyond 50
year of age. Ventriculostomy is the only other intervention that may alleviate some of the intracranial
hypertension. However, in the setting of significant neurologis deficit and osternal effacement, it is
unlikely to provide adequate relief and long term improvement in neurologic function.
--

10. What is the definition of a concussion?


A. A loss of consciousness resulting from head trauma with significant parenchymal disruption or
abnormalities
B. A loss of consciousness resulting from head trauma without significant parenchymal disruption or
abnormalities
C. A transient alteration of consciosness resulting from head trauma without significant
parenchymal disruption or abnormalities
D. A transient alteration of consciosness resulting from head trauma with significant parenchymal
disruption or abnormalities
A concussion is defined as a transient alteration of consciousness resulting from head trauma,
without significant parenchymal disruption or abnormalities. The diagnosis has been used
interchangeably with the diagnosis of Mild Traumatic Brain Injury. A period of loss of consciousness
is not required for the diagnosis.
Any loss of consciousness generally connotes a moderate to severe concussion although grading
varies. Although loss of consciousness is widely used to determine grade, post concussive amnesia
may have a stronger correlation with degree of persistent cerebral dysfunction.
--
11. What is the indication for repair of anterior wall frontal sinus fractures?
A. Acute and or chronic sinusitis
B. Formation of mucocele
C. CSF leak and resulting meningitis
D. Repair of cosmetic deformity
E. All of the above
If there is a question of cosmetic deformity in the future, the fracture may be explored for possible
reduction and function. The anterior wall of the frontal sinus is the stronger of the two tables. The
anterior wall is very rarel associated with CSF leak or injury to the drainage system. The key issues
in the management of these fractures are to determine the extent of injuries. Specifically, to determine
whether the posterior wall is involved and if there are associated injuries to the drainage system. If
the injuries are isolated to the anterior wall. The issues of infection, mucocele formation, and sinusitis
are less relevant to the management of these fractures.

1. A 37-year-old gentleman is experiencing severe pain in his left arm following a motorcycle
accident in which he suffered traumatic brachial plexus avulsion. Now two years later, the patients
continues to have a flail arm with no evidence of return of function. Medical therapy has failed to
relieve his symptoms. Which of the following is the BEST treatment for this patients pain?
A. Lesion of dorsal root entry zone
B. Neurotization of denervated segments
C. Neurectomy at affected segments
D. Implant spinal cord stimulator
E. Implant morphine pump
Dorsal root entry zone lesion successfully treats brachial plexus avultion pain 50-70% of cases in the
long term. It is one of the few ablative procedures with long term benefit in nonmalignant pain. It
should be reserved for patients who do not have any evidence of recovery of function ove rthe long
term. Neurotization procedures are unlikely to add any benefit at this late stage. In addition, they
would not treat the patients pain and are reserved for restoration to useful function. Similarly,
neurotization surgery does not addres the source of the pain which appears to be in the dorsal
rootentry zone. Intrathecal morphine does not offer any benefit, as pain of this type tends to be
unresponsive to opiates. Electrical stimulation is also unlikely to work, as it requires intregity of the
nervous system in order to facilitate conduction of the electrical impulses.
--
2. A 48 year old man who has undergone five operations on his lumbar spine aftera work-related
injury comes to your clinic complaining of severe low back pain. He is taking 300mg/day of oral
morphine is on duragesic path. He states that pain relief is inadequate with these medications and that
over the last five years his need for these medications has risen dramatically. Which of the following
BEST describes this patients behavior?
A. Narcotic addiction
B. Narcotic tolerance
C. Narcotic withdrawl
D. Malingering
Chronic use of oral opioids is almost always associated with tolerance. In some cases, the tolerance
may be therapy limiting. However, addiction-or chemical dependence- is defined as phychological
depemdence, and should be differentiated from tolerance. Umlike patients exhibiting tolerence to
opioids, chemically-dependent patients tipically seek multiple medication from multiple prescribers,
and often fail to mantain stable relationships with their physicians. Intrathecal narcotic therapy does
not obviate tolerence and certainly does not prevent addiction.
--

3. A 56-year-old man present with severe pain in the right arm secondary to extensive invasion of the
brachial plexus from a pancoast tumor. His pain extends from the shoulder region and involves the
entire arm and hand. Due to his underlying malignancy. His oncologist has estimated his life
expectancy to be less than 3 month. Which of the following represent the BEST approach for
management of his pain?
A. Spinal cord stimulation
B. C1-2 percutaneous cordotomy
C. Intrathecal opiates
D. Dorsal root entry zone lasion
E. Dorsal rhizotomy
Percutaneous C1-2 cordotomy is an excellent procedure for treatment of cancer pain located at or
below the C5 dermatomal level for patients with a limited life expecntacy. Cordotomy often
procedures immediate pain relief and often allows significant reduction in oral opiates. An
intrathecal pump could be considered, but in a patient with a life expectancy of less than 3 month,
there devices are not cost effective. Moreover, there is the requirement for ongoing maintenance and
refill of the pump. Spinal cord stimulation is not typically utilized for the management of cancer pain
and, in any event, would not be indicated in someone with such a limited life expectancy. Dorsal
rhizotomy for cancer pain is limited to cases in which the tumor has not spread into a large area. In
order to achieve effective pain relief in the entire upper extremity, a rhizotomy extending from C4
through T3 to T4 would be required, most likely leaving the patients with a non fungsional extremity.
DREZ lesioning is not customarily indicated for cancer pain.
--
4. Placement of a spinal cord stimulator lead percutaneously has been attempted but failed due to
epidural scar formation. In order to treat the patients lower extremity neuropathic pain, you decide to
perform a surgical lead placement. The most appropriate level for laminectomy is
A. C1-2
B. T10-11
C. T7-8
D. L2-3
Placement of spinal cord stimulator for relief of lower extremity pain should be performed over the
lumbal enlargement of the spianl cord, which is approximately at the T10-T11 level. This are affords
the highest likelihood of stimulating the dorsal aspect of the spinal cord with minimal stimulation of
the dorsal root. Stimulating below the tip of the spinal cord is therefore not helpful. Similary,
stimulating the spinal cord above the lumbar enlargement is likely to be associated with significant
radicular stimulation with minimal dorsal at this site may yield stimulation within the lower
extremities as well, it is less pronounced than the stimulation in the arms.
--

5. You see a patients in your clinic with complete quadriplegia secondary to spinal cord injury at C6.
He is complaining of severe rectal pain. On physical examination, he has compete anesthesia below
C7. Which one of the following is the BEST treatment option?
A. Spinal cord stimulation
B. Thoracic cordotomy
C. Deep brain stimulation
D. Amitriptyline and/or carbemazapine
E. Intrathecal morphine pump
This patients rectal pain is likely a manifestation of his spinal injury. Such pain is poorly responsive
to spinal cord stimulation. In addition, intrathecal morphine therapy is unlikely to be successful and
should not be tried in the absence of attempts at medical therapy. A surgical cordotomy does not
address midline visceral pain. The literature regarding deep brain stimulation for such pain is
relatively sparse. Use of antidepressants (such as amitriptyline) and anticonvulsants (such as
carbemazapine) is assosiated with some success in treating neurophatic pain syndromes including
those assosiated with a complete neurological injury.
1. A 15 year old high school wrestler sustains a left shoulder dislocation during a match. After the
shoulder is relocated, he has difficulty abducting his arm, although he has a strong grip and normal
sensation to pin in his hand and fingers. Your examination demonstrates normal function of the
biceps, triceps, and external rotators of the shoulder. The MOST likely explanation of these findings
is injury of :
A. Axillary nerve
B. C5 nerve root
C. Suprascapular nerve
D. Diffuse brachial plexus stretch
The patient has an isolated axillary nerve injury. This is a well recognized, though relatively
infrequent, complication of shoulder dislocation. A diffuse plexus injury is ruled out by the evidence
of motor and sensory preservation in other upper extremity muscles. The presence of shoulder
external rotatory function rules out both C5 nerve root injury and suprascapular injury.
--
2. A 38 year old male experiences severe acute right arm pain and then develops severe deltoid
wekness and mild biceps and triceps weakness. After 6 weeks he seeks consultation. Axial MRI
imaging at C4 – 5 and C5 – 6 are shown. The next appropriate step is :
A. EMG/NCS
B. Epidural steroid injection
C. Physical therapy
D. CT myelogram
E. ACDF C4 – 5 and C6 – 7

The correct answer is EMG/NS. This patientis presenting with brachial plexitis, or Parsonage –
Turner syndrome (PTS). The classic presentation of PTS is acute polyradicular pain without weakness
which resolves over two weeks, only to develop into weakness affecting multiple nerve root
distributions, most prominently the proximal arm and shoulder girdle. Significant supraspinatus and
deltoid muscle weakness result in the complete inability to abduct the shoulder, which is a hallmark of
PTS. The etiology is unknown, but preceding risk factors include recent viral illnesses, trauma or
infection. An EMG/NS could confirm multiple motor and sensory abnormalities suggesting brachial
plexus involvement and not radiculopathy, which would establish the diagnosis. ACDF is not an
appropriate next step given the likehood of PTS. The severity of weakness in the deltoid.
3. A 40 year old patient undergoes resection of a facial schwannoma that is infiltrating the facial nerve
from the cisternal segment to the mastoid segment. The nerve is resected with the tumor for a gross
total resection. When considering a facial to hypoglossal anostomosis, what is the ideal timing of
surgical intervention to maximize return of facial nerve function?
A. Within 12 months of facial nerve paralysis
B. Within 6 months of facial nerve paralysis
C. Within 2 months of facial nerve paralysis
D. Within 24 months of facial nerve paralysis
E. There is no relationship between the duration of facial nerve paralysis and functional
recovery

There is no relationship between the duration of facial nerve paralysis and functional recovery
following hypoglossal – facial nerve anastomosis (E). in comparing patients treated between 7 to 23
months as caompared to those treated within 3 months, no significant difference was identified in the
degree of facial nerve recovery. Patients can display up to a 2 month delay in functional recovery
with delayed repair. However the final results are equivalent in nearly and delayed facial nerve
repair. Patients older than 50 years have been found to have a slightly poorer but still acceptable
result as compared to younger patients.
--
4. A 65 year old man is seen for continued left medial hand pain after a failed submuscular ulnar
nerve decompression procedure two months ago. Upon re – evaluation, new (less than 1 month) left
ptosis and myosis is observed. The MOST likely diagnosis is :
A. Raeder’s Para – trigeminal neuralgia
B. Superior sulcus tumor (Pancoast tumor)
C. Parsonage Turner syndrome of the medial cord
D. Ulnar injury at an elbow level
E. Thoracic outlet syndrome
A superior sulcus tumor (a.k.a Pancoast tumor) can invade the lower trunk of the brachial plexus and
the sympathetic fibers arising from T1 (a.k.a Pancoast’s syndrome). While thoracic outlet syndrome
can affect the lower trunk, ptosis, and myosis are not characteristic findings. Raeder’s paratrigeminal
neuralgia is associated with ptosis and myosis and facial pain. It would not explain persistent medial
arm pain. Similiarly neither medial cord inflammatory injury nor ulnar nerve trauma would account
for the ptosis and myosis observed.
--
5. An 18 year old man undergoes an excisioanal biopsy of an enlarged lymph node in his posterior
cervical triangle. Followng the procedure it becomes evident that his spinal accessory nerve was
accidentally damaged. You see this patient in consultation 3 months post operatively regarding a
possible repair. Which of the folowing clinical findings is LEAST useful in confirming a trapezial
palsy secondary to spinal accessory nerve damage :
A. Ipsilateral scapular winging
B. Absent shoulder shrug
C. Ipsilateral trapezius atrophy
D. Affected shoulder lower at rest
E. Difficulty abducting affected arm > 90
The answer is absent shoulder shrug (i.e. ipsilateral scapular elevation). The ability to elevate the
scapula is usually maintained in patients with spinal accessory palsies. Spinal accessory palsies are
often undiagnosed because patients maintain their ability to shrug the affected shoulder. This is
because two muscles elevate the scapula. The upper portion of the trapezius inervated by the spinal
accessory nerve and the levator scapula innervated by motor branches from the cervical plexus.
However, this residual shoulder shrug movement is often asymmetric, which hints towards the correct
diagnosis. Furthermore, posterior cervical triangle lymph node biopsy is the most commoncause of
spinal accessory injury, and at this location some early branches to the trapezius muscle may have
been spared, which also help maintain scapular elevation. Patients with trapezius palses usually
present with scapular region discomfort and
--
6. In doing a nerve transfer procedure to recover elbow flexion which of the following two nerves are
coapted?
A. Ulnar and radial
B. Radial and musculocutaneus
C. Spinal accessory and suprascapular
D. Ulnar and musculocutaneus
E. Radial and median
This procedure is designed to re-innervate the biceps muscle for elbow flexion. It is one of the
most important of the nerve transfer procedures. In this procedure approximately 15% of the ulnar
nerves is transferred to the musculocutaneous nerve in the arm.

--
7. Lack of function in which muscle indicates a very proximal C5 nerve root injury?
A. Rhomboid
B. Flexor carpi ulnaris
C. Opponens policis
D. Supinator
E. Biceps
The dorsal scapular nerve to the rhomboid muscle is one of the few nerves to ernanate directly off a
nerve root. In this case C5. Very proximal nerve root damage or root avulsion will therefore manifest
in loss of function of this muscle (among others). A more distal injury will frequently leave the
rhomboid intact
--
8. One of the best ways to ascertain that ulnar nerve compression is distal (at the wrist) and not
proximal (at the elbow) is to
A. Test for weakness of the adductor policis muscle
B. Test the strength of the first dorsal interosseus muscle
C. Test ulnar nerve sensation on the dorsal and palmar surface of the hand
D. Test the strength of the abductor digiti minimi (V)
Testing ulnar sensation on the dorsal and palmar surfaces of the hand best distinguishes proximal
from distal ulnar neuropathy, as sensory innervations is distinct on these surfaces. The dorsal sensory
branch of the ulnar nerve leaves the ulnar nerve at approximately 2 to 3 inches proximal to the wrist
and innervates only the dorsal ulnar hand. The ulnar nerve then crosses the volar wrist through
Guyon’s canal to supply sensation to the palmar ulnar hand. Thus, ulnar nerve compression at the
wrist (e.g. Guyon’s canal syndrome) causes decreased palmar ulnar sensation and normal dorsal
ulnar sensation. All the intrinsic hand muscles may be equally affected by compression at either the
elbow or the wrist. Depending on the site of the compression within Guyon’s canal, some intrinsic
muscles may be spared
--
9. What is the best recommendation for a patient who has moderate carpal tunnel syndrome
(confirmed by EMG) but does not want surgery?
A. Steroids
B. Diuretics
C. NSAIDS
D. Botox
E. Vitamin B6
There is (1) strong evidence (level 1) on the efficacy of local and oral steroids. (2) moderate evidence
(level 2) that vitamin B6 is ineffective and splints are effective and (3) limited or conflicting evidence
(level 3) that NSAID’s, diuretics, yoga, laser and ultrasound are effective whereas exercise therapy
and botulinum toxin B injection are ineffective
--
10. Which of the following is the MOST appropriate surgical treatment for upper extremity
hyperhidrosis :
A. Carpal tunnel release
B. Vagal nerve stimulation
C. Thoracic sympathectomy
D. Dorsal rhizotomy
E. Spinal cord stimulation
Hyperhidrosis was one of the first indications for thoracic sympathectomy. T2 and T3 sympathectomy
are generally sufficient for palmar hyperhidrosis but T4 should be added in for axillary
hyperhidrosis. The surgery can be accomplished through a midline posterior incision or through a
thorascopic approach. Compensatory hyperhidrosis of other parts of the body may occur and may
limit the benefit of the surgery. The T1 sympathetic ganglia should be spared; injury to the
sympathetic ganglia at T1 may result in Horner’s syndrome.
--
11. A 40-year-old female undergoes biopsy of an enlarged posterior cervical lymph node, which
proves to be benign. Post-operatively, she develops severe aching shoulder pain on effort and loss of
muscle bulk in her supraclavicular area. The most likely diagnosis is:
A. Long Thoracic Nerve injury
B. Thoracic Outlet Syndrome

C. spinal accesory nerve injury


D. Parsonage Tumor syndrome
The cause of the problem is a spinal accessory nerve injury resulting in atrophy and weakness of the
trapezius. The shoulder pain with activity is likely due to over-compensation of the remaining
shoulder girdle muscles. Biopsy of cervical lymph nodes in the posterior triangle of the neck can
cause spinal accessory palsy by direct trauma, or stretching or division of the nerve, which runs
obliquely across the posterior triangle. Neither brachial plexitis (Parsonage-Turner syndrome) nor
cervical disc herniation affect the trapezius since the spinal accessory is a cranial nerve and not part of
the brachial plexus. The same is true of the thoracic outlet syndrome, which only involves the brachial
plexus. Long thoracic nerve palsy weakens the serratus anterior, a chest wall-based muscle and thus
does not cause supraclavicular atrophy.

You might also like